Trang chủ Đề thi & kiểm tra Lớp 12 Tiếng anh Trắc nghiệm Tiếng anh 12 Đọc hiểu có đáp án !!

Trắc nghiệm Tiếng anh 12 Đọc hiểu có đáp án !!

Câu 1 :

Read the following passage and mark the letter A, B, C, or D on your answer sheet to indicate the correct answer to each of the questions.

We get great pleasure from reading. The more advanced a man is, the greater delight he will find in reading. The ordinary man may think that subjects like philosophy or science are very difficult and that if philosophers and scientists read these subjects, it is not for pleasure.

 

But this is not true. The mathematician finds the same pleasure in his mathematics as the school boy in an adventure story. For both, it is a play of the imagination, a mental recreation and exercise. The pleasure derived from this activity is common to all kinds of reading. But different types of books give us different types of pleasure. First in order of popularity is novel-reading. Novels contain pictures of imaginary people in imaginary situations, and give us an opportunity of escaping into a new world very much like our world and yet different from it. Here we seem to live a new life, and the experience of this new life gives us a thrill of pleasure. Next in order of popularity are travel books, biographies and memoirs. These tell us tales of places we have not seen and of great men in whom we are interested.

 

Some of these books are as wonderful as novels, and they have an added value that they are true. Such books give us knowledge, and we also find immense pleasure in knowing details of lands we have not seen and of great men we have only heard of. Reading is one of the greatest enjoyments of life. To book-lovers, nothing is more fascinating than a favorite book. And, the ordinary educated man who is interested and absorbed in his daily occupation wants to occasionally escape from his drudgery into the wonderland of books for recreation and refreshment.

What does the passage mainly discuss?

A. Different types of books

B. Different kinds of reading

C. Reading as an exercise for the brain

D. Reading as a pleasurable activity

Câu 2 :
According to paragraph 1, which of the following is NOT true?

A. Ordinary people may think that philosophy and science are difficult.

B. Reading about mathematics is mentally entertaining for a mathematician.

C. Philosophers and scientists do not read for pleasure.

D. A more advanced person takes greater pleasure in reading.

Câu 3 :
The word “derived” in paragraph 2 is closest in meaning to ______.

A. differed    

B. established               

C. bought                               

D. obtained

Câu 4 :

The word “it” in paragraph 2 refers to ______.

A. a new life               

B. our world                 

C. an opportunity                   

D. a thrill of pleasure

Câu 5 :
The word “immense” in paragraph 3 is closest in meaning to ______.

A. great                      

B. limited                      

C. personal                             

D. controlled

Câu 6 :

According to the passage, travel books, biographies and memoirs ______.

A. are wonderful novels

B. tell stories of well-known places

C. are less popular than novels

D. are more valuable than novels

Câu 7 :
According to paragraph 4, which of the following is the most fascinating to booklovers?

A. A daily occupation     

B. An ordinary educated man 

C. The wonderland 

D. A favorite book

Câu 8 :

Read the following passage and mark the letter A, B, C, or D on your answer sheet to indicate the correct answer to each of the questions.

Successful students often do the followings while studying. First, they have an overview before reading. Next, they look for important information and pay greater attention to it (which often needs jumping forward or backward to process information). They also relate important points to one another. Also, they activate and use their prior knowledge. When they realize that their understanding is not good, they do not wait to change strategies. Last, they can monitor understanding and take action to correct or “fix up” mistakes in comprehension.

 

Conversely, students with low academic achievement often demonstrate ineffective study skills. They tend to assume a passive role, in learning and rely on others (e.g., teachers, parents) to monitor their studying, for example, low-achieving students often do not monitor their understanding of content; they may not be aware of the purpose of studying; and they show little evidence of looking back, or employing “fix-up” strategies to fix understanding problems. Students who struggle with learning new information seem to be unaware that they must extent effort beyond simply reading the content to understand and remember it.

 

Children with learning disabilities do not plan and judge the quality of their studying. Their studying may be disorganized. Students with learning problems face challenges with personal organization as well. They often have difficulty keeping track of materials and assignments, following directions, and completing work on time. Unlike good studiers who employ a variety of study skills in a flexible yet purposeful manner, low-achieving students use a restricted range of study skills. They cannot explain why good study strategies are important for learning; and they tend to use the same, often ineffective study approach for all learning tasks, ignoring task content, structure or difficulty.

(Source: Adapted from Study Skills: Managing Your Learning — NUI Galway)

What is the topic of the passage?


A. Successful and low-academic achieving students



B. Successful learners and their learning strategies



C. Study skills for high school students



D. Effective and ineffective ways of learning


Câu 9 :
The word “prior” in the first paragraph is closest meaning to______?

A. Important              

B. earlier                     

C. forward                             

D.   good

Câu 10 :
According to the passage, what can be learnt about passive students?

A. They depend on other people to organize their learning

B. They are slow in their studying

C. They monitor their understanding

D. They know the purpose of studying

Câu 11 :

Which of the followings is NOT an evidence of monitoring studying?

A. Being aware of the purpose of studying

B. Monitoring their understanding of content

C. Fixing up mistakes in understanding

D. Looking at their backs

Câu 12 :

According to the passage, to learn new information, low-achieving students do NOT______.

A. just understand it

B. relate it to what they have known

C. simply remember it

D. read it

Câu 13 :

In compared with low-achieving students, successful students use______.

A. aimless study techniques

B. various study skills

C. restricted strategies

D. inflexible study ways

Câu 14 :
The underlined pronoun “They” in the last sentence refers to______.

A. study strategies

B. study skills

C. low-achieving students

D. good studiers

Câu 15 :

Read the following passage and mark the letter A, B, C or D on your answer sheet to indicate the correct answer to each of the questions.

Living things include both the visible world of animals, plants, and fungi as well as the invisible world of bacteria and viruses. On a basic level, we can say that life is ordered. Organisms have an enormously complex organization. We're all familiar with the intricate systems of the basic unit of life, the cell. Life can also "work." Living creatures can take in energy from the environment. This energy, in the form of food, is transformed to maintain metabolic processes and for survival. Life grows and develops. This means more than just replicating or getting larger in size. Living organisms also have the ability to rebuild and repair themselves when injured. Life can reproduce. Think about the last time you accidentally stubbed your toe. Almost instantly, you moved back in pain. Finally, life can adapt and respond to the demands placed on it by the environment. There are three basic types of adaptations that can occur in higher organisms.

 

Reversible changes occur as a response to changes in the environment. Let's say you live near sea level and you travel to a mountainous area. You may begin to experience difficulty breathing and an increase in heart rate as a result of the change in altitude. These symptoms go away when you go back down to sea level.

 

Body-related changes occur as a result of prolonged changes in the environment. Using ther previous example, if you were to stay in the mountainous area for a long time, you would notice that your heart rate would begin to slow down and you would begin to breath normally. These changes are also reversible. Genotypic changes (caused by genetic mutation) take place within the genetic makeup of the organism and are not reversible. An example would be the development of resistance to pesticides by insects and spiders.

( Source: Adapted from http://biology.about.com/od/apforstudents/a/aa082105a.htm)

 In what way is life organized?

A. Hard                      

B. Difficult                   

C. Complicated                      

D. Problematic

Câu 16 :

Which of the following is NOT a feature of life?

A. Getting a job

B. Giving birth

C. Getting larger and left- repairing

D. Reacting to the environment

Câu 17 :
What is the energy for living things called?

A. Food                      

B. Metabolic process    

C. Green energy 

D. Environment

Câu 18 :

You see life respond most clearly when you _____.

A. intentionally hurt yourself

B. move part of your body due to threat

C. look at your toe

D. feel hurt

Câu 19 :

Which tyoe of living creatures can adapt to the changes in the environment?

A. More visible           

B. Lower                       

C. Higher                               

D. More human

Câu 20 : What does the word “reversible” in the passage mean?

A. Changeble 

B. Visible        

C. Fitful

D. Irregular

Câu 21 : Which type of adaption is permanent?

A. Body-related         

B. Reversible                

C. Genotypic     

D. Environmental

Câu 22 :

Read the following passage and mark the letter A, B, C, or D on your answer sheet to indicate the correct answer to each of the questions.

In the American colonies there was little money. England did not supply the colonies with coins and did not allow the colonies to make their own coins, except for the Massachusetts Bay Colony, which received permission for a short period in 1652 to make several kinds of silver coins. England wanted to keep money out of America as a means of controlling trade: America was forced to trade only with England if it did not have the money to buy products from other countries. The result during this pre-revolutionary period was that the colonists used various goods in place of money: beaver pelts, Indian wampum, and tobacco leaves were all commonly used substitutes for money. The colonists also made use of any foreign coins they could obtain. Dutch, Spanish, French, and English coins were all in use in the American colonies.

 

During the Revolutionary War, funds were needed to finance the world, so each of the individual states and the Continental Congress issued paper money. So much of this paper money was printed that by the end of the war, almost no one would accept it. As a result, trade in goods and the use of foreign coins still flourished during this period.

 

By the time the Revolutionary War had been won by the American colonists, the monetary system was in a state of total disarray. To remedy this situation, the new Constitution of the United States, approved in 1789, allowed Congress to issue money. The individual states could no longer have their own money supply. A few years later, the Coinage Act of 1792 made the dollar the official currency of the United States and put the country on a bimetallic standard. In this bimetallic system, both gold and silver were legal money, and the rate of exchange of silver to gold was fixed by the government at sixteen to one.

The passage mainly discusses_____.

A. the American monetary system of the seventeenth and eighteenth centuries.

B. American money from past to present.

C. the English monetary policies in colonial America.

D. The effect of the Revolution on American money.

Câu 23 :
The passage indicates that during the colonial period, money was


A. scarce freely by the colonists        

B. coined freely by the colonists

C. used extensively for trade                           

D. supplied by England

Câu 24 :

The Massachusetts Bay Colony was allowed to make coins

A. continuously from the inception of the colony   

B. for a short time during one year

C. from 1652 until the Revolutionary War               

D. throughout the seventeenth century

Câu 25 :

Which of the following is NOT mentioned in the passage as a substitute for money during the colonial period?

A. Tobacco         

B. Cotton                      

C. Wampum                           

D. Beaver furs

Câu 26 :
According to the passage, what happened to the American monetary system during the Revolutionary War?

A. The Continental Congress issued gold and silver coins.

B. American money replaced trade in goods and foreign coins.

C. Individual states were not allowed to issue money.

D. So much paper money was circulated that it lost its value

Câu 27 :

How was the monetary system arranged in the Constitution?

A. Various state governments, including Massachusetts, could issue money.

B. The dollar was made the official currency of the U.S.

C. The U.S. officially went on a bimetallic monetary system.

D. Only the U.S. Congress could issue money.

Câu 28 :

According to the passage, which of the following is NOT true about the bimetallic monetary system?

A. Gold could be exchanged for silver at a rate of 16 to 1

B. It was established in 1792

C. The monetary system was based on two metals

D. Either gold or silver could be used as official money.

Câu 29 :

Read the following passage and mark the letter A, B, C, or D on your answer sheet to indicate the correct answer to each of the questions.

Just two months after the flight of Apollo 10, the Apollo 11 astronauts made their historic landing on the surface of the Moon. This momentous trip for humanity also provided scientists with an abundance of material for study; from rock and soil samples brought back from the Moon, scientists have been able to determine much about the composition of the Moon (as well as to draw) inferences about the development of the Moon from its composition.

 

The Moon soil that came back on Apollo 11 contains small bits of rock and glass which were probably ground from larger rocks when meteors impacted with the surface of the Moon. The bits of glass are spherical in shape and constitute approximately half of the Moon soil. Scientists found no trace of animal or plant life in this soil.

 

In addition to the Moon soil, astronauts gathered two basic types of rocks from the surface of the Moon: Basalt and breccia. Basalt is a cooled and hardened volcanic lava common to the Earth. Since basalt is formed under extremely high temperatures, the presence of this type of rock is an indication that the temperature of the Moon was once extremely hot. Breccia, the other kind of rock brought back by the astronauts, was formed during the impact of falling objects on the surface of the Moon. It consists of small pieces of rock compressed together by the force of impact. Gases such as hydrogen and helium were found in some of the rocks, and scientists believe that these gases were carried to the Moon by the solar wind, the streams of gases that are constantly emitted by the Sun.

Question 29: It is implied in the passage that scientists believe that the gases found in the Moon rocks

A. were not originally from the Moon

B. caused the Moon's temperature to rise

C. traveled from the Moon to the Sun

D. were created inside the rocks

Câu 30 :
What does the word “It” refers to?

A. the impact              

B. the surface                

C. breccia                               

D. the Moon

Câu 31 :
The author's purpose in this passage is to

A. demonstrate the difference between basalt and breccia

B. explain some of the things learned from space flights

C. describe some rock and soil samples

D. propose a new theory about the creation of the Moon

Câu 32 :
According to the passage, what does Moon soil consist of?

A. Large chunks of volcanic lava.                 

B. Streams of gases

C. Tiny pieces of stones and glass.     

D. Hydrogen and helium

Câu 33 :

The word 'emitted' in the last paragraph is closest in meaning to

A. vaporized     

B. sent out                    

C. separated                           

D. set off

Câu 34 :

According to the passage, breccia was formed

A. from volcanic lava                                               

B. when objects struck the Moon

C. when streams of gases hit the surface of the Moon 

D. from the interaction of helium and hydrogen

Câu 35 :
What is the subject of this passage?

A. The Apollo astronauts                                

B. Basalt and breccia

C. What the Moon is made of                         

D. Soil on the Moon

Câu 36 :

Which of the following was NOT brought back to the Earth by the astronauts?

A. Soil 

B. Breccia       

C. Plant life     

D. Basalt

Câu 37 :

Read the following passage and mark the letter A, B, C, or D on your answer sheet to indicate the correct answer to each of the questions.

Marianne Moore (1887-1972) once said that her writing could be called poetry only because there was no other name for it. Indeed her poems appear to be extremely compressed essays that happen to be printed in jagged lines on the page. Her subjects were varied: animals, laborers, artists, and the craft of poetry. From her general reading came quotations that she found striking or insightful. She included these in her poems, scrupulously enclosed in quotation marks, and sometimes identified in footnotes. Of this practice, she wrote, "Why many quotation marks?" I am asked ... When a thing has been so well that it could not be said better, why paraphrase it? Hence, my writing is, if not a cabinet of fossils, a kind of collection of flies in amber." Close observation and concentration on detail and the methods of her poetry.

 

Marianne Moore grew up in Kirkwood, Missouri, near St. Lois. After graduation from Bryn Mawr College in 1909, she taught commercial subjects at the Indian School in Carlisle, Pennsylvania. Later she became a librarian in New York City. During the 1920’s she was editor of The Dial, an important literary magazine of the period. She lived quietly all her life, mostly in Brooklyn, New York. She spent a lot of time at the Bronx Zoo, fascinated by animals. Her admiration of the Brooklyn Dodgers-before the team moved to Los Angeles-was widely known.

 

Her first book of poems was published in London in 1921 by a group of friends associated with the Imagist movement. From that time on her poetry has been read with interest by succeeding generations of poets and readers. In 1952 she was awarded the Pulitzer Prize for her Collected Poems. She wrote that she did not write poetry for money or fame. To earn a living is needful, but it can be done in routine ways. One writes because one has a burning desire to objectify what it is indispensable to one's happiness to express.

What is the passage mainly about?

A. Essayists and poets of the 1920's.

B. The use of quotations in poetry.

C. Marianne Moore's life and work.

D. The influence of the Imagists on Marianne Moore.

Câu 38 :
Which of the following can be inferred about Moore's poems?

A. They were all published in The Dial.

B. They tend to be abstract.

C. They are better known in Europe than the United States.

D. They do not use traditional verse forms.

Câu 39 : According to the passage, Moore wrote about all of the following EXCEPT

A. artists    

B. workers                    

C. animals                              

D. fossils

Câu 40 :

What does Moore refer to as "flies in amber" (paragraph 1)?

A. Concentration on detail.

B. Poetry in the twentieth-century.

C. A common image in her poetry.

D. Quotations within her poetry.

Câu 41 : The author mentions all of the following as jobs held by Moore EXCEPT

A. librarian                 

B. commercial artist      

C. teacher                               

D. magazine editor

Câu 42 : Where did Moore spend most of her adult life?

A. In Carlisle              

B. In Kirkwood            

C. In Los Angeles                  

D. In Brooklyn

Câu 43 : The word “it” in the third paragraph refers to

A. writing poetry        

B. earning a living       

C. becoming famous               

D. attracting readers

Câu 44 :
It can be inferred from the passage that Moore wrote because she

A. was dissatisfied with what others wrote

B. wanted to win awards

C. wanted to raise money for the Bronx Zoo

D. felt a need to express herself

Câu 45 :

Read the following passage and mark the letter A, B, C, or D on your answer sheet to indicate the correct answer to each of the questions

For a century before the Erie Canal was built, there was much discussion among the general population of the Northeast as to the need for connecting the waterways of the Great Lakes with the Atlantic Ocean. A project of such monumental proportions was not going to be undertaken and completed without a supreme amount of effort.

 

The man who was instrumental in accomplishing the feat that was the Erie Canal was Dewitt Clinton. As early as 1812, he was in the nation's capital petitioning the federal government for financial assistance on the project, emphasizing what a boon to the economy of the country the canal would be; his efforts with the federal government, however, were not successful.

 

In 1816, Clinton asked the New York State Legislature for the funding for the canal, and this time he did succeed. A canal commission was instituted, and Clinton himself was made head of it. One year later, Clinton was elected governor of the state, and soon after, construction of the canal was started.

 

The canal took eight years to complete, and Clinton was on the first barge to travel the length of the canal, the Seneca Chief, which departed from Buffalo on October 26, 1825, and arrived in New York City on November 4. Because of the success of the Erie Canal, numerous other canals were built in other parts of the country.

 

 The pronoun “it” in the 3rd paragraph refers to which of the following?

A. The canal

B. The New York State Legislation

C. The state governor

D. The commission

Câu 46 :

When did Clinton ask the U.S. government for funds for the canal?

On hundred years before the canal was built.

A. In 1812  

B. In 1825      

C. In 1816

D. On hundred years before the canal was built.

Câu 47 : The Seneca Chief was

A. the nickname of Buffalo

B. the name of the canal

C. Clinton’s nickname

D. the name of a boat

Câu 48 : The information in the passage

A. is in chronological order

B. is organized spatially

C. gives a cause followed by an effect 

D. lists opposing viewpoints of a problem

Câu 49 : The word “boon” in the 2nd paragraph is closest in meaning to

A. Disadvantage        

B. Benefit                     

C. Cost                                   

D. Detriment

Câu 50 :

Where in the passage does the author mention a committee that worked to develop the canal?

A. The first paragraph

B. The second paragraph

C. The fourth paragraph

D. The third paragraph

Câu 51 :

Read the following passage and mark the letter A, B, C, or D on your answer sheet to indicate the correct answer to each of the questions.

Rain pounded down on the roof. I was trying to read but the sound was too loud. I couldn’t help myself from being a little grumpy. I wanted to be outside playing, but the rain was keeping me inside.

My mom had gone to the grocery store, and my dad was spending Saturday at the office. I had planned to spend the day hiking, but Mother Nature decided that today was the perfect day for rain.

It meant that I would have to entertain myself. I spent most of the morning playing with my stuffed animals and reading. I was sitting next to the window staring out when I got a strange idea: why not just go outside anyway?

I put on my boots and a big raincoat and stepped out into the wet world. It was raining hard but it wasn’t cold. All I could hear were raindrops and the wind. I decided to go on my hike anyway.

My feet didn’t make any sound on the wet ground and the forest seemed different. I went to my favourite place and sat down. In the summer, my best friend Ellen and I would come here and sit for hours. It was our special place. All of a sudden, I thought I heard someone shouting my name. I turned and saw Ellen walking up behind me.

“Oh my Gosh! It’s really you, Martha!” she said. “I can’t believe that you are out here right now. I thought I would be the only person crazy enough to go for a walk in the rain.”

I was very happy to have some company. We decided that hiking in the rain was just as fun as hiking in the sunshine. We planned on hiking in the rain again.

Question 1: What is the best title for the story?

A. Rainy Day Work                                        

B. Rainy Day Hike  

C. A Rainy Day Indoors                                 

D. Rainy Day Homework

Câu 52 :

The word “grumpy” in paragraph 1 is closest in meaning to

A. stupid                    

B. bad-tempered           

C. uninterested                       

D. unsatisfactory

Câu 53 :

What was keeping Martha inside?

A. The heat                 

B. Her parents               

C. Bad weather                      

D. Lots of homework

Câu 54 :
What does Martha mean when she says “It meant that I would have to entertain myself”?

A. She was tired.                                           

B. She was feeling sick

C. She had to find something to do                

D. She was bored with doing homework

Câu 55 :
The word “company in the last paragraph is closest in meaning to

A. time                       

B. space                        

C. friend                                 

D. business

Câu 56 :

What did Martha think about being outside?

A. It was too hot.       

B. It was too cold.       

C. It was very nice.     

D. It was too wet to walk.

Câu 57 :

What will Martha and Ellen probably do next time it rains?

A. Stay inside                                                 

B. Do homework

C. Go for another hike                                    

D. Go to their friend’s house

Câu 58 :

Read the following passage and mark the letter A, B, C, or D on your answer sheet to indicate the correct answer to each of the questions.

 

A large number of inventions require years of arduous research and development before they are perfected. For instance, Thomas Edison had to make more than 1,000 attempts to invent the incandescent light bulb before he finally succeeded. History is replete with numerous other examples of people trying, yet failing to make inventions before they eventually succeeded. Yet some inventions have come about not through hard work but simply by accident.

 

In most cases, when someone unintentionally invented something, the inventor was attempting to create something else. For example, in the 1930s, chemist Roy Plunkett was attempting to make a new substance that could be used to refrigerate items. He mixed some chemicals together. Then, he put them into a pressurized container and cooled the mixture. By the time his experiment was complete, he had a new invention. It was not a new substance that could be used for refrigeration though. Instead, he had invented Teflon, which is today most commonly used to make nonstick pots and pans. Similarly, decades earlier, John Pemberton was a pharmacist in Atlanta, Georgia. He was attempting to create a tonic that people could use whenever they had headaches. While he was not successful in that endeavor, he managed to invent Coca - Cola, the world - famous carbonated soft drink.

Scientists have also made crucial discoveries by accident when they were conducting experiments. In 1928, Alexander Fleming discovered penicillin, an antibiotic, in this manner. He discovered some mold growing in a dish with some bacteria. He noticed that the bacteria seemed to be avoiding the mold. When he investigated further, he determined some of the many useful properties of penicillin, which has saved millions of lives over the past few decades. Likewise, in 1946, scientist Percy Spencer was conducting an experiment with microwaves. He had a candy bar in his pocket, and he noticed that it suddenly melted. He investigated and learned the reason why that had happened. Soon afterward, he built a device that could utilize microwaves to heat food: the microwave oven.

Question 8: Which title best summarizes the main idea of the passage?

A. History's Most Important Inventions

B. Accidental Inventions and Discoveries

C. How to Become a Great Inventor

D. You Don't Always Get What You Want

Câu 59 : In paragraph 1, the word arduous is closest in meaning to _______.

A. detailed                 

B. tough                        

C. specific                              

D. constant

Câu 60 :
In paragraph 2, the word endeavor is closest in meaning to _______.

A. research                


B. dream                     



C. request                              



D. attempt


Câu 61 :

What does the author say about Teflon?

A. People first used it as a refrigeration device

B. It was created many years before Coca-Cola

C. The man who made it was a pharmacist

D. It is used for kitchenware nowadays

Câu 62 :

Who was John Pemberton?

A. The person who made Teflon

B. The creator of Coca-Cola

C. The man who discovered penicillin

D. The inventor of the microwave

Câu 63 :

The author uses Alexander Fleming as an example of ________.

A. one of the most famous inventors in history

B. a person who made an accidental scientific discovery

C. someone who became a millionaire from his invention

D. a man who dedicated his life to medical science

Câu 64 :
What does the author imply about penicillin?

A. Doctors seldom use it nowadays.

B. Some people are not affected by it.

C. It is an invaluable medical supply.

D. Mold combines with bacteria to make it.

Câu 65 :

Read the following passage and mark the letter A, B, C, or D on your answer sheet to indicate the correct answer to each of the questions.

 

In 1752, Benjamin Franklin made his textbook experiment with a brass key and a silk kite that he flew in a thunderstorm to prove that lightning and electricity are the same thing. In 1920, a kite-flying championship for families and individuals was held in London. These two seemingly unrelated events underscore the fact that kites can be flown for both pleasure and scientific purposes. For example, in the 1800s weather bureaus flew kites to record temperature and humidity at certain altitudes. On one occasion, ten kites were strung together and flown at a height of four miles to lift men and carry cameras aloft.

The kite’s ability to fly depends on its construction and the way that its line is attached. The familiar diamond - shaped kite flies when its covered face is aligned against the wind flow. The line attached to the nose of the kite pulls it into the wind, thus creating the necessary angle for the lift force. If the kite’s construction and the angle of the air stream are correct, the kite will encounter greater pressure against its face and lower pressure against its back. The difference in the pressure creates a lift that causes the kite to rise until it hangs level from its bridle. Its angle against the wind should be sufficiently large or small to create maximum lift to overcome both drag and gravity. The towing point to which the line is attached is important because it sets the kite’s angle relative to the air flow. Although the kite must be headed up and into the wind with a velocity of 8 to 20 miles per hour, it can maintain its position through a tail, a rudder, a keel, vents, or tassels.

Question 15: What is the main topic of the passage?

A. What makes kites stay up in the air.

B. How kites can be utilized.

C. What parts kites consist of.

D. Why kites are were spurned

Câu 66 :

The word “aloft” in the reading passage is closet in meaning to _____.

A. for the analysis      

B. in the flood              

C. in flight                              

D. for amusement

Câu 67 :
According to the passage, the kite flies when its nose is ______.

A. pointed away from the ground

B. balanced with the tail

C. pointed into the wind flow

D. aligned parallel to the wind flow

Câu 68 :

What is necessary condition for the kite to fly?

A. The kite must be sufficiently strong to withstand great pressure.

B. The kite must be diamond-shaped, and the wind of a certain velocity.

C. The pressure against its back must be lower than the pressure against its face.

D. The pressure of the air flow must be lower than the weight of the kite.

Câu 69 :

According to the passage, the line of the kites is important because it _____.

A. lifts the kite’s cover and frame into the air space

B. contributes to the shape of the kite and extends it

C. conveys the direction of the wind and the air flow

D. determines the angle between the kite and the air flow

Câu 70 : The phrase “headed up” in the passage is closet in meaning to ______.

A. diverted                 

B. directed                    

C. drafted                              

D. dropped

Câu 71 :

The paragraph following the passage most would likely discuss _____.

A. fiberglass kites flown in competition

B. the cords and wires needed for kite flying

C. elements of kite design and composition

D. bowed kites curved on their faces

Câu 72 :

Read the following passage and mark the letter A, B, C, or D on your answer sheet to indicate the correct answer to each of the questions.

 

The official residence of the president of the United States is the White House, located at 1600 Pennsylvania Avenue, in Washington, D.C. The Commissioners of the District of Columbia held a meeting in 1792 and decided to hold a contest for the best design for the Presidential House. James Hoban, an architect born in Ireland, was the winner. His bid for the construction of the mansion asked for $200,000, but the final cost of the building came to twice that amount. The work on the project began during the same year, and the grounds of approximately one and a half miles west of the Capitol Hill were chosen by Major Pierre-Charles L’Enfant, who was in charge of city planning. However, the construction continued for several more years, and George Washington had stepped down as president before the building was habitable. When John Adams, the second president of the United States and his wife Abigail moved in 1800, only six rooms had been completed.

 

The grey sandstone walls of the house were painted white during construction, and the color of the paint gave the building its name. The building was burned on August 24, 1814, and James Hoban reconstructed the house for President James Monroe and his family, who moved there in 1817. The north portico was added to the building in 1829, water pipes were installed in 1833, gas lighting in 1848, and electricity in 1891. In 1948, inspectors announced that the building was so dilapidated that it was beyond repair and suggested that it was cheaper to construct a new one than repair the existing dwelling. However, the national sentiment was to keep the original form intact, and Congress appropriated $5.4 million dollars for repairs. In 1961, Jacqueline Kennedy launched a program to redecorate the rooms and appointed a Fine Arts Committee to make choices of furnishing and colors.

 

The house of the president accords its residents a great deal of space. The living quarters contain 107 rooms, 40 corridors, and 19 baths. The White House contains a doctor’s suite, a dentist’s office, a large solarium, a broadcasting room, and a two-floor basement for storage and service rooms. The office in which the president works is not located in the White House, but in a separate building called the West Wing. The White House stands on 16 acres of parklike land and overlooks a broad lawn, flower gardens, and wood groves.

The word “contest” is closest in meaning to _____.

A. hearing                   

B. concourse                 

C. computation                      

D. competition

Câu 73 :

What does the passage imply about the cost of the White House construction?

A. It did not adhere to the original estimate.

B. It was proposed at the meeting of the commissioners.

C. It was not included in the architectural design.

D. It was considered excessive for the presidential home.

Câu 74 : The word “grounds” is closest in meaning to _____.

A. high ground           

B. site                            

C. several lots                         

D. hills

Câu 75 :

It can be inferred from the passage that _____.

A. George Washington contributed to the White House design

B. George Washington never lived in the White House

C. The White House was excluded from the city planning

D. George Washington often used the White House steps

Câu 76 : The author of the passage implies that the construction of the main White House building continued _____.

A. up to 1800             

B. after 1800                 

C. until 1814                          

D. until 1792

Câu 77 :

What can be inferred about the White House from the information in the second paragraph?

A. Running water was installed in the second half of the 19th century.

B. Each president added new features to the building’s conveniences.

C. Few changes occurred in the structure in the first half of the 20th century.

D. The building was modernized extensively during one decade.

Câu 78 : In line 20, the word “launched” is closest in meaning to _______.

A. stopped                  

B. worked                     


C. began                               

D. requested

Câu 79 : The passage mentions all of the following White House premises EXCEPT _____.

A. hallways                

B. kitchen                     

C. medical offices                  

D. storage rooms

Câu 80 :

Read the following passage and mark the letter A, B, C, or D on your answer sheet to Indicate the correct answer to each of the questions.

 

There is a wide range of organisms by their population whose lives mostly depend on how they hunt or are hunted. And most living organisms have some way of protecting themselves from natural predators. Some mammals, like the platypus, carry internal toxins to transmit to predators via biting or other means, and some plants protect themselves by being poisonous. The African crested rat was originally thought to be poisonous because predators that tried to eat it often became paralyzed. But scientists have recently learned that's not actually the case.

 

The crested rat chews on the poisonous bark of a certain tree, and then smears the chewed-up substance onto its fur, where a strip of special quill-like hairs soaks up the poisonous mixture. Though similar to a porcupine's, the quills do differ: whereas the porcupine defends itself by poking predators, the African rat uses its quill-like hairs to deliver poison to them. When a predator comes after it, instead of running away, the rat stays put and parts its hair to reveal the strip of fur on its back where the poison is being stored. That raised strip is the first thing that receives a bite, and the poison inside disables the predator. These hair tubes are unusual. In fact, scientists do not know of another animal that uses plant poison in this way.

 

Scientists are puzzled that the rat doesn't appear to be affected by the poison. Because it affects

heartbeat regulation, understanding how the rat can keep its heart rate regulated effectively while using the poison could help scientists develop new medicines for people with heart trouble. And they hope that those suffering from heart diseases will benefit from this.

What is the main purpose of the passage?

A. To show how animals can affect plant life.

B. To compare one animal to other similar ones.

C. To emphasize the dangers of researching animals.

D. To discuss an unusual animal defense technique.

Câu 81 :

In paragraph 1, why does the author mention the platypus?

A. To illustrate that mammals may eat poisonous plants.

B. To give an example of a mammal that uses poison.

C. To show how it differs from the crested rat.

D. To explain that the platypus is at risk.

Câu 82 : The word “paralyzed” in paragraph 1 is closest in meaning to

A. disabled                 

B. unaffected                

C. inaccessible                        

D. undecided

Câu 83 : In paragraph 2, what does the word “them” refer to?

A. predators               

B. porcupines                

C. crested rats                        

D. quill-like hairs

Câu 84 : The word “defends” in paragraph 2 is closest in meaning to

A. encourages             

B. protects                    

C. feeds                                  

D. pretends

Câu 85 : What are scientists surprised by?

A. How toxic plant poison can be.

B. How useful plant toxins can be.

C. How quickly rats learn new behavior.

D. That the rats are protected from the poison.

Câu 86 : What would scientists like to investigate further?

A. Medicine used to counteract poison

B. Other animals like the crested rat.

C. Health effects of a chemical.                      

D. Rats with unhealthy hearts.

Câu 87 :

Read the following passage and mark the letter A, B, C, or D on your answer sheet to indicate the correct answer to each of the questions.

The end of the nineteenth century and the early years of the twentieth century were marked by the development of an international Art Nouveau style, characterized by sinuous lines, floral and vegetable motifs, and soft evanescent coloration. The Art Nouveau style was an eclectic one, bringing together elements of Japanese art, motifs of ancient cultures, and natural forms. The glass objects of this style were elegant in outline, although often deliberately distorted, with pale or iridescent surfaces. A favored device of the style was to imitate the iridescent surface seen on ancient glass that had been buried. Much of the Art Nouveau glass produced during the years of its greatest popularity had been generically termed “art glass.” Art glass was intended for decorative purposes and relied for its effect on carefully chosen color combinations and innovative techniques.

France produced a number of outstanding exponents of the Art Nouveau style; among the most celebrated was Emile Galle (1846-1904). In the United States, Louis Comfort Tiffany (1843-1933) was the most noted exponent of this style, producing a great variety of glass forms and surfaces, which were widely copied in their time and are highly prized today. Tiffany was a brilliant designer, successfully combining ancient Egyptian, Japanese, and Persian motifs.

 

The Art Nouveau style was a major force in the decorative arts from 1895 until 1915, although its influence continued throughout the mid-1920’s.It was eventually to be overtaken by a new school of thought known as Functionalism that had been present since the turn of the century. At first restricted to a small avant-garde group of architects and designers, Functionalism emerged as the dominant influence upon designers after the First World War. The basic tenet of the movement-that function should determine form-was not a new concept. Soon a distinct aesthetic code evolved: from should be simple, surfaces plain, and any ornament should be based on geometric relationships. This new design concept, coupled with the sharp postwar reactions to the styles and conventions of the preceding decades, created an entirely new public taste which caused Art Nouveau types of glass to fall out of favor. The new taste demanded dramatic effects of contrast, stark outline and complex textural surfaces.

 What does paragraph 1 mainly discuss?

A. Design elements in the Art Nouveau style

B. The popularity of the Art Nouveau style

C. Production techniques for art glass       

D. Color combinations typical of the Art Nouveau style

Câu 88 : The word “one” refers to______.

A. century.                 

B. development.           

C. style.                                  

D. coloration.

Câu 89 : Para.1 mentions that Art Nouveau glass was sometimes similar to which aspect of ancient buried glass______.

A. The distortion of the glass

B. The appearance of the glass surface

C. The shapes of the glass objects                   

D. The size of the glass objects

Câu 90 : The word “overtaken” in line 19 is closest in meaning to______.

A. surpassed               

B. inclined                  

C. expressed                             

D. applied

Câu 91 :

What does the author mean by stating that “function should determine form”?

A. A useful object should not be attractive.

B. The purpose of an object should influence its form.

C. The design of an object is considered more significant than its function.

D. The form of an object should not include decorative elements.

Câu 92 :

It can be inferred from the passage that one reason Functionalism became popular was that it______.

A. clearly distinguished between art and design

B. appealed to people who liked complex painted designs

C. reflected a common desire to break from the past

D. was easily interpreted by the general public

Câu 93 :

Paragraph 3 supports which of the following statements about Functionalism?

A. Its design concept avoided geometric shapes.

B. It started on a small scale and then spread gradually.

C. It was a major force in the decorative arts before the First World War.

D. It was not attractive to architects all designers.

Câu 94 : According to the passage, an object made in the Art Nouveau style would most likely include ______.

A. a flowered design.     

B. bright colors.       

C. modern symbols.               

D. a textured surface.

Câu 95 :

Read the following passage and mark the letter A, B, C, or D on your answer sheet to indicate the correct answer to each of the questions.

Telecommuting is some form of computer communication between employees’ homes and offices. For employees whose job involve sitting at a terminal or word processor entering data or typing reports, the location of the computer is of no consequence. If the machine can communicate over telephone lines, when the work is completed, employees can dial the office computer and transmit the material to their employers. A recent survey in USA Today estimates that there are approximately 8,7 million telecommuters. But although the numbers are rising annually, the trend does not appear to be as significant as predicted when Business Week published “The Portable Executive” as its cover story a few years ago. Why hasn’t telecommuting become more popular?

Clearly, change simply takes time. But in addition, there has been active resistance on the part of many managers. These executives claim that supervising the telecommuters in a large work force scattered across the country would be too difficult, or, at least, systems for managing them are not yet developed, thereby complicating the manager’s responsibilities.

It is also true that employees who are given the option of telecommuting are reluctant to accept the opportunity. Most people feel that they need regular interaction with a group, and many are concerned that they will not have the same consideration for advancement if they are not more visible in the office setting. Some people feel that even when a space in their homes is set aside as a work area, they never really get away from the office.

 With which of the following topics is the passage primarily concerned?

A. An overview of telecommuting.       

B. The failure of telecommuting.

C. The advantages of telecommuting.             

D. A definition of telecommuting.

Câu 96 : How many Americans are involved in telecommuting?

A. More than predicted in Business Week

B. More than 8 million.

C. Fewer than last year.

D. Fewer than estimated in USA Today

Câu 97 : The phrase “of no consequence” means

A. of no use                

B. irrelevant                  

C. of no good                         

D. unimportant

Câu 98 : The author mentions all of the following as concerns of telecommuting, EXCEPT

A. the opportunities for advancement

B. the different system of supervision

C. the lack of interaction with a group.           

D. the work place is in the home.

Câu 99 : The word “them” in the second paragraph refers to

A. telecommuters           

B. systems               

C. executives                          

D. responsibilities

Câu 100 :

The reason why telecommuting has not become popular is that the employees

A. need regular interaction with their families.

B. are worried about the promotion if they are not seen at the office.

C. feel that a work area in their home is away from the office.

D. are ignorant of telecommuting.

Câu 101 : The word “reluctant” in the third paragraph can best be replaced by

A. opposite                 

B. willing                      

C. hesitant                              

D. typical

Câu 102 :

Read the following passage and mark the letter A, B, C or D on your answer sheet to indicate the correct answer to each of the questions.

Biological diversity has become widely recognized as a critical conservation issue only in the past two decades. The rapid destruction of the tropical rain forests, which are the ecosystems with the highest known species diversity on Earth, has awakened people to the importance and fragility of biological diversity. The high rate of species extinctions in these environments is jolting, but it is important to recognize the significance of biological diversity in all ecosystems. As the human population continues to expand, it will negatively affect one after another of Earth‟s ecosystems. In terrestrial ecosystems and in fringe marine ecosystems (such as wetlands), the most common problem is habitat destruction. In most situations, the result is irreversible. Now humans are beginning to destroy marine ecosystems through other types of activities, such as disposal and runoff of poisonous waste; in less than two centuries, by significantly reducing the variety of species on Earth, they have irrevocably redirected the course of evolution.

Certainly, there have been periods in Earth‟s history when mass extinctions have occurred. The extinction of the dinosaurs was caused by some physical event, either climatic or cosmic. There have also been less dramatic extinctions, as when natural competition between species reached an extreme conclusion. Only 0.01 percent of the species that have lived on Earth have survived to the present, and it was largely chance that determined which species survived and which died out.

However, nothing has ever equaled the magnitude and speed with which the human species is altering the physical and chemical world and demolishing the environment. In fact, there is wide agreement that it is the rate of change humans are inflicting, even more than the changes themselves, that will lead to biological devastation. Life on Earth has continually been in flux as slow physical and chemical changes have occurred on Earth, but life needs time to adapt-time for migration and genetic adaptation within existing species and time for the proliferation of new genetic material and new species that may be able to survive in new environments.

What does the passage mainly discuss?

A. The cause of the extinction of the dinosaurs

B. The variety of species found in tropical rain forests

C. The time required for species to adapt to new environments

D. The impact of human activities on Earth‟s ecosystems

Câu 103 : The word “critical” is closest in meaning to

A. complicated           

B. interesting                          

C. negative                   

D. essential

Câu 104 :
The author mentions the extinction of the dinosaurs in the 2nd paragraph to emphasize that?

A. the cause of the dinosaurs‟ extinction is unknown

B. actions by humans could not stop the irreversible process of a species‟ extinction

C. not all mass extinctions have been caused by human activity

D. Earth‟s climate has changed significantly since the dinosaurs‟extinction

Câu 105 : The word “jolting” is closest in meaning to

A. shocking                

B. unknown                            

C. illuminating              

D. predicted

Câu 106 :  The author mentions all of the following as examples of the effect of humans on the world‟s

A. habitat destruction in wetlands

B. the introduction of new varieties of plant species

C. damage to marine ecosystems      

D. destruction of the tropical rain forests

Câu 107 : With which of the following statements would the author be most likely to agree?

A. Human influence on ecosystems should not be a factor in determining public policy.

B. Humans should be more conscious of the influence they have on ecosystems.

C. The extinction of a few species is an acceptable consequence of human progress.

D. Technology will provide solutions to problems caused by the destruction of ecosystems.

Câu 108 :

Read the following passage and mark the letter A, B, C, or D on your answer sheet to indicate the correct answer to each of the questions.

William Sydney Porter (1862-1910), who wrote under the pseudonym of O. Henry, was born in North Carolina. His only formal education was to attend his Aunt Lina‟s school until the age of fifteen, where he developed his lifelong love of books. By 1881 he was a licensed pharmacist. However, within a year, on the recommendation of a medical colleague of his Father‟s, Porter moved to La Salle County in Texas for two years herding sheep. During this time, Webster‟s Unabridged Dictionary was his constant companion, and Porter gained a knowledge of ranch life that he later incorporated into many of his short stories. He then moved to Austin for three years, and during this time the first recorded use of his pseudonym appeared, allegedly derived from his habit of calling “Oh, Henry” to a family cat. In 1887, Porter married Athol Estes. He worked as a draftsman, then as a bank teller for the First National Bank.

In 1894 Porter founded his own humor weekly, the “Rolling Stone”, a venture that failed within a year, and later wrote a column for the Houston Daily Post. In the meantime, the First National Bank was examined, and the subsequent indictment of 1886 stated that Porter had embezzled funds. Porter then fled to New Orleans, and later to Honduras, leaving his wife and child in Austin. He returned in 1897 because of his wife‟s continued ill-health, however she died six months later. Then, in 1898 Porter was found guilty and sentenced to five years imprisonment in Ohio. At the age of thirty five, he entered prison as a defeated man; he had lost his job, his home, his wife, and finally his freedom. He emerged from prison three years later, reborn as O. Henry, the pseudonym he now used to hide his true identity. He wrote at least twelve stories in jail, and after re-gaining his freedom, went to New York City, where he published more than 300 stories and gained fame as America‟s favorite short Story writer. Porter married again in 1907, but after months of poor health, he died in New York City at the age of forty-eight in 1910. O. Henry‟s stories have been translated all over the world.

 According to the passage, Porter‟s Father was _____.

A. the person who gave him a life-long love of books

B. a medical doctor

C. a licensed pharmacist

D. responsible for his move to La Salle County in Texas

Câu 109 : Why did the author write the passage?

A. to outline the career of a famous American

B. because of his fame as America‟s favorite short story writer

C. because it is a tragic story of a gifted writer

D. to outline the influences on O. Henry‟s writing

Câu 110 : The word “imprisonment” in paragraph 2 is closet in meaning to ____________.

A. captivity                 

B. escape                                

C. insult                        

D. punishment

Câu 111 : What is the passage primarily about?

A. The life and career of William Sydney Porter

B. The way to adopt a nickname.

C. O.Henry‟s influence on American literature.        

D. The adventures of O.Henry.

Câu 112 : The author implies which of the following is true?

A. Porter‟s wife might have lived longer if he had not left her in Austin when he fled.

B. Porter was in poor health throughout his life.

C. O. Henry is as popular in many other countries as he is in America.

D. Porter would probably have written less stories if he had not been in prison for three years.

Câu 113 : Which of the following is true, according to the passage?

A. Porter left school at 15 to become a pharmacist

B. Porter wrote a column for the Houston Daily Post called “Rolling Stone”

C. The first recorded use of his pseudonym was in Austin

D. Both of Porter‟s wives died before he died

Câu 114 : The word “pseudonym” in the passage refers to _____.

A. William Sydney Porter           

B. O. Henry             

C. Athol Estes            

D. the Aunt Lina

Câu 115 :

Read the following passage and mark the letter A, B, C, or D on your answer sheet to indicate the correct answer to each of the questions.

The Forbidden City is the former imperial palace in the center of Beijing, China. Construction began in 1406, and the emperor's court officially moved in by 1420. The Forbidden City got its name because most people were barred from entering the 72-hectare site, surrounded by walls. Even government officials and the imperial family were permitted only limited access. Only the emperor could enter any section at will.

The architecture of the Forbidden City conforms rigidly to traditional Chinese principles. All buildings within the walls follow a north-south line and the most important ones face south to honor the sun. The designers arranged the other buildings, and the ceremonial spaces between them, to impress all visitors with the great power of the Emperor, while reinforcing the insignificance of the individual. This architectural concept was carried out to the smallest detail. For example, the importance of a building was determined not only by its height or width but also by the style of its roof and the quantity of statuettes placed on the roof‟s ridges.

In recognition of the importance of its unparalleled architecture, UNESCO added the palace to its World Heritage List in 1987. Today, visitors from all over the world do not wait for an imperial invitation to walk about this palace, now a museum of imperial art.

One of the most impressive landmarks of the Forbidden City is the Meridian Gate, the formal entrance to the southern side of the Forbidden City. The gate, with its auxiliary wings on either side of the entryway, is 38 meters high at its roof ridge. When you stand in front of this majestic structure, you understand how awed people felt when they stood there listening to imperial proclamations.

As you walk through the gate, you come into a large courtyard, 140 meters long and 210 meters wide. Running through the courtyard is the Golden River, which is crossed by five parallel white marble bridges. These bridges lead to the Gate of Supreme Harmony, which, in turn, leads to the heart of the Forbidden City. At the northernmost end of the Forbidden City is the Imperial Garden, which is totally different from the rest of the compound. Instead of rigid formality, you see a seemingly spontaneous

According to the passage, what do the bridges over the Golden River lead to?

A. The Meridian gate


B. The Gate of Supreme Harmony


C. The Imperial Gardens 

D. The center of Beijing arrangement of trees, fishponds, flowerbeds, and sculpture. Here is the place of relaxation for the emperor.

Câu 116 :

Which phrase is closest in meaning to the word 'unparalleled' as used in paragraph 3?

A. high quality found nowhere else               


B. at an angle from the main line

C. careless of small details in design               

D. partially designed in a foreign country

Câu 117 :

From the passage, it is implied that the main entrance area to the Forbidden City is

A. not very impressive                                    


B. surrounded by three tall walls

C. decorated with statuettes                            

D. painted gold and green

Câu 118 :

Which word(s) does the word 'its' refer to in paragraph 3?

A. Architecture          

B. World Heritage List          

C. Palace                       

D. UNESCO

Câu 119 : Which word or phrase is closest in meaning to the word "spontaneous”?

A. Without thinking   

B. Unique                               

C. Without planning     

D. Strange

Câu 120 :

From the passage, it can be inferred that

A. Chinese architects borrowed ideas from many different countries

B. the garden of the Forbidden City was laid out in a strict, rectangular pattern.

C. the design of the Forbidden City is dull and colorless.

D. the architecture of the Forbidden City exemplifies traditional Chinese values.

Câu 121 :

Which phrase is closest in meaning to the word “proclamations”?

A. Poetry written for the emperor.      

B. Music composed for public ceremonies.

C. Speeches encouraging soldiers to fight.               

D. Official public announcements.

Câu 122 : Which sentence in the first paragraph explains who could go anywhere in the Forbidden City at any time?

A. Sentence 2             

B. Sentence 5                         

C. Sentence 4               

D. Sentence 3

Câu 123 :

. Read the following passage and mark the letter A, B, C, or D on your answer sheet to indicate the correct answer to each of the questions.

People have been donating blood since the early twentieth century to help accident victims and patients undergoing surgical procedures. Usually a pint of whole blood is donated, and it is then divided into platelets, white blood cells, and red blood cells. People can donate blood (for red blood cells) about once every two months. Transfusing the blood from the donor to the recipient is straightforward. It involves taking the blood from a donor‟s arm vein by means of a hypodermic syringe. The blood flows through a plastic tube to a collection bag or bottle that contains sodium citrate, which prevents the blood from clotting.

When the blood is given to a patient, a plastic tube and hypodermic needle are connected to the recipient's arm. The blood flows down from the container by gravity. This is a slow process and may last as long as 2 hours to complete the infusion of blood into the recipient. The patient is protected from being infected during the transfusion. Only sterile containers, tubing, and needles are used, and this helps ensure that transfused or stored blood is not exposed to disease causing bacteria. Negative reactions to transfusions are not unusual. The recipient may suffer an allergic reaction or be sensitive to donor leukocytes. Some may suffer from an undetected red cell incompatibility. Unexplained reactions are also fairly common. Although they are rare, other causes of such negative reactions include contaminated blood, air bubbles in the blood, overloading of the circulatory system through administration of excess blood, or sensitivity to donor plasma or platelets. Today, hospitals and blood banks go to great lengths to screen alt blood donors and their blood. All donated blood is routinely and rigorously tested for diseases, such as HIV, hepatitis B, and syphilis.

When the recipient is a newborn or an infant, the blood is usually irradiated to eliminate harmful elements. Donated blood is washed, and the white blood cells and platelets removed. Storing the blood sometimes requires a freezing process. To freeze the red blood cells, a glycerol solution is added. To unfreeze the, the glycerolis removed. The ability to store blood for long periods has been a boon to human health

 All of the following are mentioned as potential negative reactions to transfusion EXCEPT

A. Sensitivity to donor leukocytes                            

B. Air bubbles in the blood

C. Allergies     

D. Red-cell incompatibility

Câu 124 : What can the phrase "go to great length" best replaced by?

A. irradiated               

B. rigorously                           

C. routinely                   

D. removed

Câu 125 : According to the passage, how often can people donate blood for red blood cells?

A. Every four months 

B. Every three month            

C. Every two months   

D. Every month

Câu 126 : Based on the information in the passage, what can be inferred about blood transfused to infants and newborns?

A. It is treated with radiant energy.

B. It is not treated differently from adults.

C. It is not dangerous for children.                           

D. It is rigorously tested as blood for adults.

Câu 127 :

The word "it" refers to

A. surgical procedures                                               

B. accident victims       

C. a pint of whole blood                                           

D. surgery patients

Câu 128 : What answer choice is closest in meaning to the word “undetected”?

A. not illustrated        

B. not captured                      

C. not found                 

D. not wanted

Câu 129 : What does the author imply in the passage?

A. Motoring blood benefits mankind

B. Clotting cannot be prevented.

C. Freezing blood destroys platelets.                        

D. Transfusing blood is a dangerous process.

Câu 130 :

Read the following pasage and mark the letter A, B, C or D on your answer sheet to indicate the correct answer to each of the questions

Any list of the greatest thinkers in history contains the name of the brilliant physicist Albert Einstein. His theories of relativity led to entirely new ways of thinking about time, space, matter, energy, and gravity. Einstein's work led to such scientific advances as the control of atomic energy, even television as a practical application of Einstein's work.

In 1902 Einstein became an examiner in the Swiss patent office at Bern. In 1905, at age 26, he published the first of five major research papers. The first one provided a theory explaining Brownian movement, the zig-zag motion of microscopic particles in suspension. The second paper laid the foundation for the photon, or quantum, theory of light. In it he proposed that light is composed of separate packets of energy, called quanta or photons, that have some of the properties of particles and some of the properties of waves. A third paper contained the "special theory of relativity" which showed that time and motion are relative to the observer, if the speed of light is constant and the natural laws are the same everywhere in the universe. The fourth paper was a mathematical addition to the special theory of relativity. Here Einstein presented his famous formula, E = m(cc), known as the energy mass equivalence. In 1916, Einstein published his general theory of relativity. In it he proposed that gravity is not a force, but a curve in the space-time continuum, created by the presence of mass.

Einstein spoke out frequently against nationalism, the exalting of one nation above all others. He opposed war and violence and supported Zionism, the movement to establish a Jewish homeland in Palestine. When the Nazis came to power in 1933, they denounced his ideas. He then moved to the United States. In 1939 Einstein learned that two German chemists had split the uranium atom. Einstein wrote to President Franklin D. Roosevelt warning him that this scientific knowledge could lead to Germany developing an atomic bomb. He suggested the United States begin its own atomic bomb research.

 According to the passage l, Einstein's primary work was in the area of

A. chemistry               

B. biology                               

C. physics                     

D. math

Câu 131 : Which of the following inventions is mentioned in the passage as a practical application of Einstein's discoveries?

A. Radio                     

B. Television                             

C. Automobiles          

D. Computer

Câu 132 :

According to the passage, Einstein supported all of the following except


A. nationalism                                  



B. the establishment of a Jewish homeland in Palestine



C. atomic bomb research in the United States.          


D. the defeat of the Nazis.

Câu 133 : According to the passage in which country did Einstein live in 1930s?

A. Switzerland           

B. the United States               

C. Germany                  

D. Israel

Câu 134 : The word “exalting” in the passage is closest in meaning to

A. criticism                 

B. elimination                         

C. suppor                      

D. elevation

Câu 135 :

It is clear from the tone of the passage that the author feels


A. Einstein's work in physics was somewhat tarnished by his conservative political views.



B. Einstein's work in physics, though theoretically impressive, led to few practical applications.



C. Albert Einstein was one of the most brilliant thinkers in history.


D. Einstein's theories have been consistently proven incorrect.

Câu 136 :

Read the following pasage and mark the letter A, B, C or D on your answer sheet to indicate the correct answer to each of the questions.

As heart disease continues to be the number-one killer in the United States, researchers have become increasingly interested in identifying the potential risk factors that trigger heart attacks. High-fat diets and "life in the fast lane" have long been known to contribute to the high incidence of heart failure. But according to new studies, the list of risk factors may be significantly longer and quite surprising.

Heart failure, for example, appears to have seasonal and temporal patterns. A higher percentage of heart attacks occur in cold weather, and more people experience heart failure on Monday than on any other day of the week. In addition, people are more susceptible to heart attacks in the first few hours after waking. Cardiologists first observed this morning phenomenon in the mid-1980, and have since discovered a number of possible causes. An early-morning rise in blood pressure, heart rate, and concentration of heart stimulating hormones, plus a reduction of blood flow to the heart, may all contribute to the higher incidence of heart attacks between the hours of 8:00 A.M. and 10:00 A.M.

In other studies, both birthdays and bachelorhood have been implicated as risk factors. Statistics reveal that heart attack rates increase significantly for both females and males in the few days immediately preceding and following their birthdays. And unmarried men are more at risk for heart attacks than their married counterparts. Though stress is thought to be linked in some way to all of the aforementioned risk factors, intense research continues in the hope of further comprehending why and how heart failure is

triggered.

Question 1: What does the passage mainly discuss?

A. Risk factors in heart attacks                

B. Seasonal and temporal pattern of heart attacks

C. Cardiology in the 1980s     

D. Diet and stress as factors in heart attacks

Câu 137 :

The word “trigger” as used in the first paragraph is closest in meaning to which of the following?

A. involve                   

B. affect                      

C. cause                       

D. encounter

Câu 138 :

What do the second and the third paragraphs of the passage mainly discuss?

A. The link between heart attacks and marriage       

B. Unusual risk factors in heart attacks

C. Age and gender factors in heart attacks             

D. Myths about lifestyles and heart attacks

Câu 139 :

The phrase “susceptible to” in the second paragraph could best be replaced by

A. aware of              

B. affected by             

C. accustomed to        

D. prone to

Câu 140 :

According to the passage, which of the following is NOT a possible cause of any heart attacks?

A. Decreased blood flow to the heart            

B. Increased blood pressure

C. Lower heart rate                                          

D. Increase in hormones

Câu 141 :

Which of the following is NOT cited as a possible risk factor?

A. Having a birthday                          

B. Getting married

C.  Eating fatty foods            

D. Being under stress

Câu 142 :

Which of the following does the passage infer?

A. We now fully understand how risk factors trigger  heart attacks.

B. We recently began to study how risk factors trigger heart attacks

C. We have not identified many risk factors associated with heart attacks.

D. We do not fully understand how risk factors trigger heart attacks.

Câu 143 :

Read the following passage and mark letter A, B, C, or D on your answer sheet to indicate the correct answer to each of the questions

Until recently, hunting for treasure from shipwrecks was mostly fantasy; with recent technological advances, however, the search for sunken treasure has become more popular as a legitimate endeavor. This has caused a debate between those wanting to salvage the wrecks and those wanting to preserve them.

Treasure hunters are spurred on by the thought of finding caches of gold coins or other valuable objects on a sunken ship. One team of salvagers, for instance, searched the wreck of the RMS Republic, which sank outside the Boston harbor in 1900. The search party, using side-scan sonar, a device that projects sound waves across the ocean bottom and produces a profile of the sea floor, located the wreck in just two and a half days. Before the use of this new technology, such searches could take months or years. The team of divers searched the wreck for two months, finding silver tea services, crystal dinnerware, and thousands of bottles of wine, but they did not find the five and a half tons of American Gold Eagle coins they were searching for.

Preservationists focus on the historic value of a ship. They say that even if a shipwreck's treasure does not have a high monetary value, it can be an invaluable source of historic artifacts that are preserved in nearly mint condition. But once a salvage team has scoured a site, much of the archaeological value is lost. Maritime archaeologists who are preservationists worry that the success of salvagers will attract more treasure-hunting expeditions and thus threaten remaining undiscovered wrecks. Preservationists are lobbying their state lawmakers to legally restrict underwater searches and unregulated salvages. To counter their efforts, treasure hunters argue that without the lure of gold and million-dollar treasures, the wrecks and their historical artifacts would never be recovered at all.

Question 8: What is the main idea of this passage?

A. Searching for wrecks is much easier with new technologies  like side-scan sonar.

B. Maritime archaeoclogists are concerned about the unregulated searching of wrecks.

C. The search of the RMS Republic failed to produce the hoped-for coins.

D. The popularity of treasure seeking has spurred a debate between preservationists and salvagers.

Câu 144 :

The word “sunken” is closest in meaning to which of the following words?

A. broken                   

B. underwater              

C. ancient                               

D. hollow

Câu 145 :

Which of the following statements is best supported by the author?

A. The value of a shipwreck depends on the quantity of its artifacts.

B. Preservationists  are fighting the use of technological  advances such as side-scan sonars

C. Side-scan sonar has helped to legitimize salvaging.

D. The use of sound waves is crucial to locating shipwrecks.

Câu 146 :
The author uses the word “services” to refer to which of the following?

A. cups                       

B. sets                           

C. containers                          

D. decorations

Câu 147 :

All of the following were found on the RMS Republic EXCEPT

A. wine bottles                                               

B. silver tea services  

C. American Gold Eagle coins                        

D. crystal dinnerware

Câu 148 :
From the passage, you can infer that a preservationist would be most likely to

A. shun treasure-seeking salvagers                 

B. be a diver

C. put treasures in a museum                          

D. do archaeological research

Câu 149 :

The word “scoured” is most similar to which of the following?

A. scraped away      

B. scratched over          

C. scrambled around             

D. Searched through

Câu 150 :

The second and third paragraphs are an example of

A. chronological order   

B. explanation            

C. specific to general              

D. definition

Câu 151 :

Read the following passage and write the letter A, B, C or D on the top of the first page to indicate the correct answer to each of the questions.

The incredible growth of the Internet over recent years has caused problems for parents and teachers. Parents worry about which sites their children spend time on, who they chat to online and the possible effects that computer games might have on them. For teachers, meanwhile, the main worry is the way the Internet makes cheating easier!

Schools and universities say there has been a huge increase in plagiarism – taking other people‟s words and ideas and pretending that they are your own. In the past, anyone who wanted to copy had to go to a library, find the right books, read through them, find the sections they needed and then physically write down the words they wanted to use. Nowadays, though, students can simply copy extracts from websites- while really desperate students sometimes copy whole essays! As if this wasn‟t bad enough, sites offering to actually do homework – at a price – have also started appearing.

Despite all this, we shouldn‟t assume that the Internet only brings problems. Indeed, you could say that for every problem the Internet creates, it also brings a solution. Parents can now use sophisticated controls to stop kids accessing sites that might do them harm, while new software helps teachers to detect copied work immediately. Many, of course, are already able to recognize when someone is cheating! „Some students suddenly start using words they can‟t possibly understand like „dialectical antagonism‟,‟ explains one teacher, „or parts of their essays feel different.”

One of the hardest things for teachers today is deciding how to mix modern technology with traditional study skills – and how best to use the Web in class. As more and more schools install computers in every classroom, the role of the teacher is changing. Making sure students don‟t just copy things and do learn how to quote copied work properly is part of their job, but so is designing suitable projects to fully exploit the Web in helping students students learn about subjects and develop their life and social skills.

( adapted from Pre- Intermidiate Outcomes by Hugh Dellar $ Andrew Walkley)

Question 16: What is the passage mainly about?

A. Students wasting time on computer games

B. The Interner making cheating easier

C. The effect that the Web is having on school life

D. The Web used in projects to teach social skills

Câu 152 :
The word “them” in paragraph 1 refers to ______.

A. parents                   

B. children                    

C. games                                

D. teachers

Câu 153 :

According to the passage, which of the following statements is NOT true?

A. Plagiarism is copying someone‟s work and pretending it‟s your own.

B. Designin projects to take advantage of the Internet is one of parents‟ duties.

C. Many teachers can recognize  when their students are cheating.

D. Parents can use some software to stop children assessing harmful sites.

Câu 154 :

The word “this” in paragraph 2 refers to _____.

A. copying other people‟s work                    

B. finding the right  books

C. sale of homework on the Web                    

D. physically writing down words

Câu 155 :

According to paragraph 3, teachers can recognize plagiarism because ______.

A. there is always a solution to the problem.

B. students leave clues from the original

C. the Internet makes parents and teachers worried.

D. students use perfect English, above their level.

Câu 156 :
The word “ detect” in paragraph 3 is closest in meaning to ____.

A. ignore                   

B. notice                      

C. admire                               

D. confide

Câu 157 :
Which of the following can be inferred from the passage?

A. The Internet has only bad impacts on education.

B. Many websites offer to do homework without  fee.

C. The Internet can be used positively in education.

D. Parents don‟t want to control children‟s access to sites.

Câu 158 :

Read the following passage and mark the letter A, B, C or D on your answer sheet to

indicate the correct answer to each of the questions.

Most of the early houses built in America were suited to farm life, as it was not until cities became manufacturing centers that colonists could survive without farming as their major occupation. Among the earliest farmhouses in America were those built in Plymouth Colony. Generally they consisted of one large rectangular room on the ground floor, called a hall or great room and having a fireplace built into one of the walls, and a loft overhead. Sometimes a lean-to was attached alongside the house to store objects such as spinning wheels, firewood, barrels, and tubs. The furnishings in the great room were sparse and crudely built. Tabletops and chest boards were split or roughly sawed and often smoothed only on one side. Benches took the place of chairs, and the table usually had a trestle base so it could be dismantled when extra space was required. One or two beds and a six-board chest were located in one corner of the room. The fireplace was used for heat and light, and a bench often placed nearby for children and elders, in the area called the inglenook.

The original houses in Plymouth Colony were erected within a tall fence for fortification. However, by 1630 Plymouth Colony had 250 inhabitants, most living outside the enclosure. By 1640, settlements had been built some distance from the original site. Villages began to emerge throughout Massachusetts and farmhouses were less crudely built. Windows brought light into homes and the furnishings and décor were more sophisticated.

As more diversified groups of immigrants settled the country, a greater variety of farmhouses appeared, from Swedish long-style houses in the Delaware Valley to saltbox houses in Connecticut, Dutch-Flemish stone farmhouses in New York, and clapboard farmhouses in Pennsylvania. From Georgian characteristics to Greek revival elements, farmhouses of varied architectural styles and building functions populated the landscape of the new frontier.

Question 23: The main idea of the passage is  

A. The history of the American farmhouse    

B. Where immigrants settled in America

C. How to build an American farmhouse       

D. life in Plymouth Colony

Câu 159 :

Which of the following is not mentioned as part of the furnishings in farmhouses?

A. Rocking chair        

B. Bench                       

C. Trestle- based table           

D. Six – board chest

Câu 160 :

According to the passage the earliest farmhouses were built in

A. Delaware Valley    

B. Massachusetts          

C. Connecticut                       

D. Pennsylvania

Câu 161 :

The word "it" in paragraph 1 refers to

A. trestle base             

B. space                        

C. table                                   

D. chest board

Câu 162 :

It can be inferred from the passage that

A. The major occupation in Plymouth Colony was carpentry

B. Sophisticated tools were available to the early immigrants

C. Cloth was important from England

D. the extended family lived together in the farmhouse

Câu 163 :
The passage was most probaly written by a specialist in American

A. urban planning    

B. farming                     

C. architecture                      

D. immigration

Câu 164 :
The word emerge” in paragraph 2 could best be replaced with

A. proceed                  

B. settle                         

C. come out                            

D. appear

Câu 165 :

Read the following passage and mark the letter A, B, C, or D on your answer sheet to indicate the correct answer to each of the questions.

One of the seven wonders of the ancient world, the Great Pyramid of Giza was a monument of wisdom and prophecy built as a tomb for Pharaoh Cheops in 2720 B.C. Despite its antiquity, certain aspects of its construction makes it one of the truly wonders of the world. The thirteen- acre structure near the Nile river is a solid mass of stone blocks covered with limestone. Inside are the number of hidden passageways and the burial chamber of the Pharaoh. It is the largest single structure in the world. The four sides of the pyramid are aligned almost exactly on true north, south, east and west-an incredible engineering feat. The ancient Egyptians were sun worshippers and great astronomers, so computations for the Great Pyramid were based on astronomical observations.

Explorations and detailed examinations of the base of the structure reveal many intersecting lines. Further scientific study indicates that these represent a type of timeline of events – past, present and future. Many of the events have been interpreted and found to coincide with known facts of the past. Others are prophesied for future generations and are currently under investigation. Many believe that pyramids have supernatural powers and this one is no exception. Some researchers even associate it with extraterrestrial beings of ancient past.

Was this superstructure made by ordinary beings, or one built by a race far superior to any known

today?

Question 30: In the second passage, the word, ‘prophesied’ is closest in meaning to ____.

A. foretold                 

B. terminated                

C. precipitated                      

D. affiliated

Câu 166 :

On what did the ancient Egyptians base their calculations?

A. Advanced tools of measurement           

B. Knowledge of the earth‟s surface

C. Advanced technology                                 

D. Observation of the celestial bodies

Câu 167 :

What was the most probable reason for providing so many hidden passages?

C. To keep grave robbers from finding the tomb and the treasure buried with the Pharaoh.

D. To enable the Pharaoh‟s family to bring food for his journey to the afterlife

Câu 168 :

The word feat’ in the first paragraph is closet in meaning to ____.

A. accomplishment     

B. festivity                   

C. appendage                         

D. structure

Câu 169 :

What is the best title for the passage?

A. Problems with the Construction of the Great Pyramid

B. Exploration of the Burial Chamber of Cheops

C. Symbolism of the Great Pyramid

D. Wonders of the Great Pyramid of Giza

Câu 170 :

Why is the Great Pyramid of Giza considered one of the seven wonders of the world?

A. It was built by a super race.

B. It is perfectly aligned  with the four cardinal points of the compass and contains many prophecies.

C. It was selected of the tomb of Pharaoh Cheops.

D. It was very old.

Câu 171 :

What has research of the base revealed?

A. There are cracks in the foundation             

B. Tomb robbers have stolen the Pharaoh‟s body

C. A superior race of people built in                

D. The lines represent important events

Câu 172 :

Read the passage and mark A, B, C or D to indicate the correct answer to each of the questions.

Jupiter is the largest and most massive planet and is fifth in order of distance from the sun. It is well placed for observation for several months in every year and on average is the brightest of the planets apart from Venus, though for relatively brief periods Mars may outshine it. Jupiter‟s less than 10 hour rotation period gives it the shortest day in the solar system in so far as the principal planets are concerned. There are no true seasons on Jupiter because the axial inclination to the perpendicular of the orbital plane is only just over 3°-less than that for any other planet.

The most famous mark on Jupiter is the Great Red Spot. It has shown variations in both intensity and color, and at times it has been invisible, but it always returns after a few years. At its greatest extent it may be 40,000 kilometers long and 14,000 kilometers wide, so its surface area is greater than that of Earth. Though the latitude of the Red Spot varies little, it drifts about in longitude. Over the past century the total longitudinal drift has amounted to approximately 1200°. The latitude is generally very close to -22°. It was once thought that the Red Spot might be a solid or semisolid body floating in Jupiter‟s outer gas. However, the Pioneer and Voyager results have refuted that idea and proven the Red Spot to be a phenomenon of Jovian meteorology. Its longevity may well due to its exceptional size, but there are signs that it is decreasing in size, and it may not be permanent. Several smaller red spots have been seen occasionally but have not lasted.

Question 37: It can be inferred from the passage

A. a day on Earth is shorter than a day on Jupiter

B. there are other structures on Jupiter that has the same size as the Great Red Spot

C. there are times when Great Red Spot cannot be observed from the earth

D. the Great Red Spot is the only structure on Jupiter

Câu 173 :

According to the passage, which planet typically shines the most brightly?

A. Earth          

B. Jupiter         

C. Venus         

D. Mars

Câu 174 :

According to the passage, the Great Red Spot________.

A. has different  colors                                   

B. is as big as the earth

C. is a solid structure floating in the air           

D. has increased its size over the years

Câu 175 :

The word “it” in paragraph 2 refers to ________.

A. Jupiter                    

B. The Great Red Spot 

C. intensity                            

D. color

Câu 176 :
The word “exceptional” in paragraph 2 mostly means _______

A. extreme                  

B. sustainable                

C. temporary                         

D. infrequent

Câu 177 :
According to the passage, which of the following is NOT true?

A. Jupiter is bigger than all the other planest in the solar system.

B. A day in Jupiter is nearly 10 hours long.

C. the Red Great Spot moves vertically than horizontally.

D. Scientists have proof showing that smaller red spots are increasing their size to become other Great Red Spots.

Câu 178 :
The passage was probably taken from_______

A. an art journal                                              


B. a geology magazine


C. a high school textbook                               

D. an archaeology Book

Câu 179 :

Read the following passage and mark the letter A, B, C, or D on your answer sheet to indicate the correct answer to each of the questions.

Although the “lie detectors” are being used by governments, police departments, and businesses that all want guaranteed ways of detecting the truth, the results are not always accurate. Lie detectors are properly called emotion detectors, for their aim is to measure bodily changes that contradict what a person says. The polygraph machine records changes in heart rate, breathing, blood pressure, and the electrical activity of the skin (galvanic skin response, or GSR). In the first part of the polygraph test, you are electronically connected to the machine and asked a few neutral questions (“What is your name?”, “Where do you live?”). Your physical reactions serve as the standard (baseline) for evaluating what comes next. Then you are asked a few critical questions among the neutral ones (“When did you rob the bank?”). The assumption is that if you are guilty, your body will reveal the truth, even if you try to deny it. Your heart rate, respiration, and GSR will change abruptly as you respond to the incriminating questions.

That is the theory; but psychologists have found that lie detectors are simply not reliable. Since most physical changes are the same across all emotions, machines cannot tell whether you are feeling guilty, angry, nervous, thrilled, or revved up form an exciting day. Innocent people may be tense and nervous about the whole procedure. They may react physiologically to a certain word (“bank”) not because they robbed it, but because they recently bounced a check. In either case the machine will record a “lie”. The reverse mistake is also common. Some practiced liars can lie without flinching, and others learn to beat the machine by tensing muscles or thinking about an exciting experience during neutral questions.

Question 44: What is the main idea of this passage?


A. Lie detectors distinguish different  emotions 

B. Physical reaction reveal guilty

C. Lie detectors make innocent  people nervous       

D. How lie detectors are used and their reliability


 

Câu 180 :
According to the test, polygraph ________.

A. measure a person‟s thoughts                    

B. always reveal the truth about a person

C. make guilty people angry                           

D. record a person‟s physical reactions

Câu 181 :

According to the passage, what kind of questions is asked on the first part of the polygraph test?

A. incriminating         

B. critical                      

C. emotional                           

D. unimportant

Câu 182 :
The word “ones” in paragraph 1 refers to ______.

A. questions    

B. reactions     

C. standards   

D. evaluations

Câu 183 :
The word “it” in paragraph 1 refers to ______.

A. the question           

B. your body                 

C. the assumption                 

D. the truth

Câu 184 :
The word “assumption” in paragraph 1 could best be replaced with _____.

A. belief                     

B. faith                         

C. statement                           

D. imagining

Câu 185 :
This passage was probably written by a specialist in _____.

A. sociology               

B. anthropology            

C. criminal psychology         

D. mind reading

Câu 186 :

According to Einstein, light is composed of separate packets of energy called

A. electrons                

B. photoelectrons                   

C. gamma rays            

D. quanta

Câu 187 :

Read the following passage and mark the letter A, B, C, or D to indicate the correct answer to each of the questions.

BASKETBALL

Although he created the game of basketball at the YMCA in Springfield, Massachusetts, Dr. James A. Naismith was a Canadian. Working as a physical education instructor at the International YMCA, now Springfield College, Dr. Naismith noticed a lack of interest in exercise among students during the wintertime. The new England winters were fierce, and the students balked at participating in outdoor activities. Naismith determined a fast-moving game that could be played indoors would fill a void after the baseball and football seasons had ended.

First, he attempted to adapt outdoor games such as soccer and rugby to indoor play, but he soon found them unsuitable for confined areas. Finally, he determined that he would have to invent a game.

In December of 1891, Dr. Naismith hung two old peach baskets at either end of the gymnasium at the school, and, using a soccer ball and nine players on each side, organized the first basketball game. The early rules allowed three points for each basket and made running with the ball violation. Every time a goal was made, someone had to climb a ladder to retrieve the ball.

Nevertheless, the game became popular. In less than a year, basketball was being played in both the United States and Canada. Five years later, a championship tournament was staged in New York City, which was won by the Brooklyn Central YMCA.

The teams had already been reduced to seven players, and five became standard in 1897 season. When basketball was introduced as a demonstration sport in the 1904 Olympic Games in St. Luis, it quickly spread throughout the world. In 1906, a metal hoop was used for the first time to replace the basket, but the name basketball has remained.

Question 36: What does this passage mainly discuss?

A. The Olympic Games in St Louis in 1904            

B. The development of basketball

C. The YMCA athletic program                               

D. Dr. James Naismith

Câu 188 :
When was the first demonstration game of basketball held during the Olympics?

A. 1891                        

B. 1892                                   

C. 1897                                   

D. 1904

Câu 189 :

The phrase "balked at" in the first paragraph could best be replaced by

A. resisted                  

B. enjoyed                              

C. excelled at               

D. was exhausted by

Câu 190 :

The word "them" in the second paragraph refers to

A. indoors                  

B. seasons                              

C. games                       

D. areas

Câu 191 :
The word "retrieve" in the third paragraph is closest in meaning to

A. reclaim                  

B. get back                             

C. get along                 

D. reform

Câu 192 :

Why did Naismith decide to invent basketball?

A. He did not like soccer or rugby.

B. He was tired of baseball and football.

C. He wanted his students to exercise during the winter.

D. He could not convince his students to play indoors

Câu 193 :

The author mentions all of the following as typical of the early game of basketball EXCEPT

A. three points were scored for every basket.

B. running with the ball was not a foul.

C. nine players were on a team.         

D. the ball had to be retrieved from the basket after each score.

Câu 194 :

Read the following passage and mark the letter A, B, C or D on your answer sheet to indicate the correct answer to each of the questions.

FIRST TIME IN THE AIR

When John Mills was going to fly in an aeroplane for the first time, he was frightened. He did not like the idea of being thousands of feet up in the air. “ „I also didn‟t like the fact that I wouldn‟t be in control,” says John. “I‟m a terrible passenger in the car. When somebody else is driving, I tell them what to so. It drives everybody crazy.”

However John couldn‟t avoid flying any longer. It was the only way he could visit his grandchildren in Canada. “I had made up my mind that I was going to do it, I couldn‟t let my son, his wife and their three children travel all the way here to visit me. It would be so expensive for them and I know Tom‟s business isn‟t doing so well at the moment – it would also be tiring for the children – it‟s a nine-hour flight!” he says. To get ready for the flight John did lots of reading about aeroplanes. When he booked his seat, he was told that he would be flying on a Boeing 747, which is better known as a jumbo jet. “I needed to know as much as possible before getting in that plane. I suppose it was a way of making myself feel better. The Boeing 747 is the largest passenger aircraft in the world at the moment. The first one flew on February 9th 1969 in the USA. It can carry up to 524 passengers and 3.400 pieces of luggage. The fuel for aeroplanes is kept in the wings and the 747‟s wings are so big that they can carry enough fuel for an average car to be able to travel 16,000 kilometres a year for 70 years. Isn‟t that unbelievable? Even though I had discovered all this very interesting information about the jumbo, when I saw it for the first time, just before I was going to travel to Canada, I still couldn‟t believe that something so enormous was going to get up in the air and fly. I was even more impressed when I saw how big it was inside with hundreds of people!”

The biggest surprise of all for John was the flight itself. “The take-off itself was much smoother than I expected although I was still quite scared until we were in the air. In the end, I managed to relax, enjoy the food and watch one of the movies and the view from the window was spectacular. I even managed to sleep for a while! Of course,” continues John, “the best reward of all was when I arrived in Canada and saw my son and his family, particularly my beautiful grandchildren. Suddenly, I felt so silly about all the years when I couldn‟t even think of getting on a plane. I had let my fear of living stop me from seeing the people I love most in the world. I can visit my son and family as often as I like now!” Question 43: Why did John Mills fly in an aeroplane?

A. He wanted to go on holiday                                

B. He wanted to try it.

C. He wanted to see his family                                 

D. He had to travel on business.

Câu 195 :

Why did John read about aeroplane?

A. He wanted to know how they work.                 

B. It was his hobby.

C. It made him feel safer.       

D. He had found a book on them

Câu 196 :

What happened when he saw the jumbo jet for the first time?


A. He felt much safer.    

B. He liked the shape of it.

C. He couldn‟t believe how big it was.          

D. He thought the wings were very small

Câu 197 :

How did John feel when the aeroplane was taking off?

A. excited                   

B. happy                                 

C. sad

D. fear

Câu 198 :

What surprised John most about the flight?

A. that he liked the food.                                     

B. that he was able to sleep

C. that there was a movie being shown                    

D. that the view was good

Câu 199 :

How did John feel about his fears in the end?

A. He thought he had wasted time being afraid.         

B. He realized it was okay to be afraid.

C. He hoped his grandchildren weren‟t afraid of flying.    

D. He realized that being afraid kept him safe.

Câu 200 :

Read the following passage and mark the letter A, B, C, or D on your answer sheet to indicate the correct answer to each of the questions.

Pollution is a threat to many species on Earth, but sometimes it can cause species to thrive. Such is the case with Pfiesteria piscicida. A one-celled creature called a dinoflagellate, Pfiesteria inhabits warm coastal areas and river mouths, especially along the eastern United States. Although scientists have found evidence of Pfiesteria in 3,000-year-old sea floor sediments and dinoflagellates are thought to be one of the oldest life forms on earth, few people took notice of Pfiesteria.

Lately, however, blooms – or huge, dense populations – of Pfiesteria are appearing in coastal waters, and in such large concentrations the dinoflagellates become ruthless killers. The blooms emit powerful toxins that weaken and entrap fish that swim into the area. The toxins eventually cause the fish to develop large bleeding sores through which the tiny creatures attack, feasting on blood and flesh. Often the damage is astounding. During a 1991 fish kill, which was blamed on Pfiesteria on North Carolina’s Neuse River, nearly one billion fish died and bulldozers had to be brought in to clear the remains from the river. Of course, such events can have a devastating effect on commercially important fish, but that is just one way that Pfiesteria causes problems. The toxins it emits affect human skin in much the same way as they affect fish skin. Additionally, fisherman and others who have spent time near Pfiesteria blooms report that the toxins seem to get into the air, where once inhaled they affect the nervous system, causing severe headaches, blurred vision, nausea, breathing difficulty, short-term memory loss and even cognitive impairment.

For a while, it seemed that deadly Pfiesteria blooms were a threat only to North Carolina waters, but the problem seems to be spreading. More and more, conditions along the east coast seem to be favorable for Pfiesteria. Researchers suspect that pollutants such as animal waste from livestock operations, fertilizers            washed from farmlands and waste water from mining operations have probably all combined to promote the growth of Pfiesteria in coastal waters.

What is true of Pfiesteria?

A. It seems to flourish in the presence of certain pollutants.

B. It has been a menace to fish and humans for over 3000 years.

C. It is the oldest life form on earth

D. In large concentrations, it poses a threat to fish but not to humans.

Câu 201 :
What is the main function of the toxins emitted by the dinoflagellates?

A. They are quick-acting poisons that kill fish within minutes.

B. They weaken the fish just long enough for the tiny creatures to attack

C. They damage the nervous system of potential predators.

D. They cause fish to develop wounds on which creatures feed.

Câu 202 :

The word “astounding” in the passage is closest in meaning to _______.

A. continual                

B. incredible                 

C. spectacular                             

D. apprehensive

Câu 203 :

What were bulldozers used for in the Neuse River?

A. cleaning up the sediment at the bottom of the river

B. excavating holes to bury the dead fish

C. scooping up the vast number of dead fish in the water 

D. removing the huge amount of Pfiesteria from the river

Câu 204 :

According to the paragraph 2, what will NOT happen if one breathes the toxic air?

A. vomiting                

B. visual impairments   

C. circulatory difficulty         

D. terrible headaches

Câu 205 :

What is especially worrying about Pfiesteria blooms?

A. Conditions are becoming increasingly favourable for their spread

B. They are fatal to humans who come in contact with them

C. They have devastated the fishing industry in U.S coastal waters 

D. Researchers have no idea as to exactly what causes them

Câu 206 :

All of the following are true, according to the passage, EXCEPT _____ 

A. Pfiesteria caused the death of about one billion fish in the late 1990s

B. animal and chemical waste from farmlands, livestock and mining operations may contribute to the expansion of Pfiesteria

C. Pfiesteria was not commonly noticed despite scientific findings

D. the toxic subtances emitted by Pfiesteria have a similar effect on human and fish skins

Câu 207 :

In which environment would you NOT expect a Pfiesteria bloom to develop?

A. a marsh which absorbs waste water from a nearby pig farm

B. a river located near a rock quarry

C. a cool mountain lake teeming with fish

D. a river that flows through rich farmland

Câu 208 :

Read the following passage and mark the letter A, B, C or D on your answer sheet to indicate the correct answer to each of the following questions

Simply being bilingual doesn’t qualify someone to interpret. Interpreting is not only a mechanical process of converting one sentence in language A into the same sentence in language B. Rather, its a complex art in which thoughts and idioms that have no obvious counterparts from tongue to tongue _ or words that have several meanings must be quickly transformed in such a way that the message is clearly and accurately expressed to the listener.

At one international conference, an American speaker said, “You cant make a silk purse out of a sows ear”, which meant nothing to the Spanish audience. The interpretation was, “A monkey in a silk dress is still a monkey” _ an idiom the Spanish understood and that expressed the same idea.

There are 2 kinds of interpreters, simultaneous and consecutive. The former, sitting in a separated booth, usually at a large multilingual conference, speaks to listeners wearing headphones, interpreting what a foreign language speaker says _ actually a sentence behind. Consecutive interpreters are the ones most international negotiations use. They are employed for smaller meetings without sound booths and headphones. Consecutive interpretation also requires two-person teams. A foreign speaker says his piece while the interpreter, using a special shorthand, takes notes and during a pause, tells the client what was said.

What is the purpose of the passage?

A. To differentiate between simultaneous and consecutive interpreters.

B. To state the qualifications of an interpreter.

C. To point out the importance of an interpreter.

D. To explain the scope of interpreting.

Câu 209 :

What is a difference mentioned between a simultaneous interpreter and a consecutive

interpreter?

A. The size of group with whom they work.  

B. Their proficiency in the language.

C. The type of dictionary they use. 

D. The money they are paid.

Câu 210 :

The word “converting” is closest in meaning to…

A. changing                

B. concluding               

C. understanding                   

D. reading

Câu 211 :

The author implies that most people have the opinion that the skill of interpreting is …..

A. very complex and demanding                   

B. based on principles of business

C. simpler than it really is                         

D. highly valued and admired

Câu 212 :

The phrase “the former“ refers to…

A. simultaneous interpreters   

B. the conference        

C. consecutive interpreters     

D. the booth

Câu 213 :

The example “You cant make a silk purse out of a sows ear” is used to...

A. point out the difference in attributes of animals in English and Spanish 

B. emphasize the need for translation of the meaning of what is said

C. show the differences in language A and language B 

D. stress the importance of word for word translation

Câu 214 :

The word “rather” is closest in meaning to….

A. in brief                    

B. on the contrary        

C. in general                           

D. as a result

Câu 215 :

Which of the following would a consecutive interpreter be used for?

A. An interpretation of a major literary work.

B. A business transaction between 2 foreign speakers.

C. A large meeting of many nations.

D. A translation of a foreign book.

Câu 216 :

Read the following passage and mark the letter A, B, C or D on your answer sheet to indicate the correct answer to each of the following questions

Fish that live on the sea bottom benefit by being flat and hugging the contours. There are two very different types of flatfish and they have evolved in very separate ways. The skates and rays, relatives of the sharks have become flat in what might be called the obvious way. Their bodies have grown out sideways to form great “wings” They look as though they have been flattened but have remained symmetrical and “the right way up”. Conversely fish such as plaice, sole, and halibut have become flat in a different way. There are bony fish which have a marked tendency to be flattened in a vertical direction; they are much “taller” than they are wide. They use their whole vertically flattened bodies as swimming surfaces, which undulate through the water as they move. Therefore when their ancestors migrated to the seabed, they lay on one side than on their bellies. However, this raises the problem that one eye was always looking down into the sand and was effectively useless - In evolution this problem was solved by the lower eye “moving” around the other side. We see this process of moving around enacted in the development of every young bony flatfish. It starts life swimming near the surface, and is symmetrical and vertically flattened, but then the skull starts to grow in a strange asymmetrical twisted fashion, so that one eye for instance the left, moves over the top of the head upwards, an old Picasso - like vision. Incidentally, some species of 20 flatfish settle on the right side, others on the left, and others on either side.

The passage is mainly concerned with:

A. symmetrical flatfish                                  

B. bony flatfish

C. evolution of flatfish                                    

D. different types of flatfish

Câu 217 :

The author mentions skates and rays as examples of fish that ____

A. become asymmetrical                                  

B. appear to fly

C. have spread horizontally                             

D. resemble sharks

Câu 218 :

It can be inferred from the passage that the early life of a flatfish is____

A. often confusing     

B. pretty normal            

C. very difficult                     

D. full of danger

Câu 219 :

It can be inferred from the passage that horizontal symmetrical fish _____


A. have one eye each side of the head



B. have one eye underneath the head


C. have two eyes on top of the head

D. have eyes that move around the head

Câu 220 :

The word “conversely” is closest in meaning to:

A. Similarly                

B. Alternatively            

C. Inversely                            

D. Contrarily

Câu 221 :

The word “this” refers to_____

A. the migration of the ancestors

B. the practice of lying on one side

C. the problem of the one eye looking downwards

D. the difficulty of the only one eye being useful

Câu 222 : According to the passage, the ability of a bony flatfish to move its eyes around is____

A. average                   

B. weak                        

C. excellent                            

D. variable

Câu 223 : Read the following passage and mark the letter A, B, C or D on your answer sheet ton indicate the correct answer to each of the questions.

A. It is the smallest of the Earth's three layers.

B. It is thicker on land than it is under the water.

C. There many separate pieces that make it up.

D. The mantle beneath it keeps it from moving too much.

Câu 224 :

The word "perceive" in bold in paragraph 3 is closest in meaning to___________.

A. detect                     

B. comprehend             

C. prevent                              

D. locate

Câu 225 :

What is the passage mainly about?

A. When earthquakes are the most likely to happen.

B. What kind of damage natural disasters

C. How earthquakes and tsunamis occur can cause.

D. Why tsunamis are deadlier than earthquakes.

Câu 226 :

The word "adjoining" in bold in paragraph 2 is closest in meaning to___________.

A. residing                  

B. approaching              

C. bordering                           

D. appearing

Câu 227 :

Which of the following is NOT mentioned in paragraph 3 about earthquakes?

A. How severe the majority of them are

B. How often powerful ones take place

C. How many people they typically kill

D. What kind of damage they can cause

Câu 228 :

 Which of the following statements does paragraph 1 support?

A. A tsunami happens in tandem with an earthquake.

B. Earthquakes cause more destruction than tsunamis.

C. The most severe type of natural disaster is an earthquake.

D. Earthquakes frequently take place after tsunamis do.

Câu 229 :

The word "it" in bold in paragraph 2 refers to___________.

A. the mantle              

B. the crust                   

C. the Earth                            

D. the core

Câu 230 :

Based on the passage, what is probably true about tsunamis?

A. They are able to move as fast as the speed of sound.

B. They cannot damage ships sailing on the ocean.

C. They can be deadly to people standing near shore.

D. They kill more people each year than earthquakes.

Câu 231 :

Read the following passage and mark the letter A, B, C, or D on your answer sheet to indicate the correct answer to each of the questions

Since water is the basis of life, composing the greater part of the tissues of all living things, the crucial problem of desert animals is to survive in a world where sources of flowing water are rare. And since man’s inexorable necessity is to absorb large quantities of water at frequent intervals, he can scarcely comprehend that many creatures of the desert pass their entire lives without a single drop.

Uncompromising as it is, the desert has not eliminated life but only those forms unable to withstand its desiccating effects. No moist- skinned, water-loving animals can exist there. Few large animals are found. The giants of the North American desert are the deer, the coyote, and the bobcat. Since desert country is open, it holds more swift-footed running and leaping creatures than the tangled forest. Its population is largely nocturnal, silent, filled with reticence, and ruled by stealth. Yet they are not emaciated.

Having adapted to their austere environment, they are as healthy as animals anywhere else in the word. The secret of their adjustment lies in the combination of behavior and physiology. None could survive if, like mad dogs and Englishmen, they went out in the midday sun; many would die in a matter of minutes. So most of them pass the burning hours asleep in cool, humid burrows underneath the ground, emerging to hunt only by night. The surface of the sun-baked desert averages around 150 degrees, but 18 inches down the temperature is only 60 degrees.

Question 32: The author mentions all the following as examples of the behavior of desert animals EXCEPT

A. they are watchful and quiet                      

B. they sleep during the day

C. they dig home underground                      

D. they are noisy and aggressive

Câu 232 :

We can infer from the passage that

A. healthy animals live longer lives              

B. living things adjust to their environment

C. desert life is colorful and diverse              

D. water is the basis of desert life

Câu 233 :

According to the passage, creatures in the desert

A. are smaller and fleeter than forest animals

B. are more active during the day than those in the tangled forest

C. live in an accommodating environment

D. are not as healthy as those anywhere else in the world

Câu 234 :
The phrase “those forms” in the passage refers to all of the following EXCEPT

A. moist-skinned animals                                

B. many large animals

C. water-loving animals                                   

D. the coyote and the bobcat

Câu 235 :

The word “them” means

A. animals                  

B. minutes                     

C. people                                

D. water

Câu 236 :

The word “emaciated” in the passage mostly means

A. wild                       

B. unmanageable          

C. cunning                              

D. unhealthy

Câu 237 :
Man can hardly understand why many animals live their whole life in the desert, as

A. water is an essential part of his existence

B. very few large animals are found in the desert

C. sources of flowing water are rare in a desert

D. water composes the greater part of the tissues of living things

Câu 238 :

The title for this passage could be

A. “Man’s Life in a Desert Environment”                

B. “Desert Plants”

C. “Animal Life in a Desert Environment”    

D. “Life Underground

Câu 239 :

Read the following passage and mark the letter A, B, C, or D on your answer sheet to indicate the correct answer to each of the following questions.

According to anthropologists, people in pre-industrial societies spent 3 to 4 hours per day or about 20 hours per week doing the work necessary for life. Modern comparison of the amount of work performed per week, however, began with the Industrial Revolution (1760-1840) when 10 to 12-hour workdays with six workdays per week were the norm. Even with extensive time devoted to work, however, both incomes and standards of living were low. As incomes rose near the end of the Industrial Revolution, it became increasingly common to treat Saturday afternoons as a half-day holiday. The half- day holiday had become standard practice in Britain by the 1870s, but did not become common in the United States until the 1920s.

In the United States, the first third of the twentieth century saw the workweek move from 60 hours per week to just under 50 hours by the start of the 1930s. In 1914 Henry Ford reduced daily work hours at his automobile plants from 9 to 8. In 1926 he announced that henceforth his factories would close for the entire day on Saturday. At the time, Ford received criticism from other firms such as United States Steel and Westinghouse, but the idea was popular with workers.

The Depression years of the 1930s brought with them the notion of job sharing to spread available work around; the workweek dropped to a modem low for the United States of 35 hours. In 1938 the Fair Labor Standards Act mandated a weekly maximum of 40 hours to begin in 1940, and since that time the 8-hour day, 5-day workweek has been the standard in the United States. Adjustments in various places, however, show that this standard is not immutable. In 1987, for example, German metalworkers struck for and received a 37.5-hour workweek; and in 1990 many workers in Britain won a 37-hour week. Since 1989, the Japanese government has moved from a 6 to a 5-day workweek and has set a national target of 1,800 work hours per year for the average worker. The average amount of work per year in Japan in 1989 was 2,088 hours per worker, compared to 1,957 for the United States and 1,646 for France.

Question 40: What does the passage mainly discuss?

A. Why people in preindustrial societies worked few hours per week

B. Changes that have occurred In the number of hours that people work per week

C. A comparison of the number of hours worked per year in several industries

D. Working conditions during the Industrial Revolution

Câu 240 :

Compared to preindustrial times, the number of hours in the workweek in the nineteenth century

A. remained constant                                      

B. decreased slightly

C. decreased significantly                               

D. increased significantly

Câu 241 :

The word "henceforth" in line 11 is closest in meaning to

A. in the end              

B. for a brief period      

C. from that time on 

D. on occasion

Câu 242 :

The "idea" mentioned in line 13 refers to

A. the 60-hour workweek

B. the reduction in the cost of automobiles

C. the reduction in the workweek at some automobile factories

D. the criticism of Ford by United States Steel and Westinghouse

Câu 243 :

What is one reason for the change in the length of the workweek for the average worker in the United States during the 1930's?

A. Several people sometimes shared a single job.

B. Labor strikes in several countries influenced labor policy in the United states.

C. Several corporations increased the length of the workweek.

D. The United States government instituted a 35-hour workweek

Câu 244 :

Which of the following is mentioned as one of the purposes of the Fair Labor Standards Art of 1938?

A. To discourage workers from asking for increased wages

B. To establish a limit on the number of hours in the workweek

C. To allow employers to set the length of the workweek for their workers

D. To restrict trade with countries that had a long workweek

Câu 245 :

The word "immutable" in line 18 is closest in meaning to

A. unmatched             

B. irregular                    

C. unnecessary                       

D. unchangeable

Câu 246 :

Read the following passage and mark the letter A, B, C, or D on your answer sheet to indicate the correct answer to each of the following questions

One of the most important social developments that helped to make possible a shift in thinking about the role of public education was the effect of the baby boom of the 1950's and 1960's on the schools. In the 1920's, but especially in the Depression conditions of the 1930's, the United States experienced a declining birth rate – every thousand women aged fifteen to forty-four gave birth to about 118 live children in 1920, 89.2 in 1930, 75.8 in 1936, and 80 in 1940. With the growing prosperity brought on by the Second WorldWar and the economic boom that followed it, young people married and established households earlier and began to raise larger families than had their predecessors during the Depression. Birth rates rose to 102 per thousand in 1946, 106.2 in 1950, and 118 in 1955. Although economics was probably the most important determinant, it is not the only explanation for the baby boom. The increased value placed on the idea of the family also helps to explain this rise in birth rates. The baby boomers began streaming into the first grade by the mid-1940's and became a flood by 1950. The public school system suddenly found itself “overtaxed”. While the number of schoolchildren rose because of wartime and postwar conditions, these same conditions made the schools even less prepared to cope with the flood. The wartime economy meant that few new schools were built between 1940 and 1945. Moreover, during the war and in the boom times that followed large numbers of teachers left their profession for better-paying jobs elsewhere in the economy.

Therefore, in the 1950's and 1960's, the baby boom hit an antiquated and inadequate school system. Consequently, the "custodial rhetoric" of the 1930's and early 1940's no longer made sense; that is, keeping youths aged sixteen and older out of the labor market by keeping them in school could no longer be a high priority for an institution unable to find space and staff to teach younger children aged five to sixteen. With the baby boom, the focus of educators and of laymen interested in education inevitably turned toward the lower grades and back to basic academic skills and discipline. The system no longer had much interest in offering nontraditional, new, and extra services to older youths.

Question 47: What does the passage mainly discuss?

A. Birth rates in the United States in the 1930s and 1940s

B. The impact of the baby boom on public education

C. The role of the family in the 1950's and 1960's

D. The teaching profession during the baby boom

Câu 247 :

The word "it" in paragraph 1 refers to____.

A. the economic boom    

B. the Second World War        

C. the 1930s          

D. the United States

Câu 248 :

The word "overtaxed" in paragraph 1 is closest in meaning to____.

A. charged too much                                      

B.plentifully supplied

C. heavily burdened                                        

D. well prepared

Câu 249 :
The public school of the 1950s and 1960s faced all of the following problems EXCEPT

A. an inadequate number of school buildings

B. old-fashioned facilities

C. a shortage of teachers                                 

D. a declining number of students

Câu 250 :

According to the passage, why did teachers leave the teaching profession after theoutbreak of the war?

A. Teaching positions were scarce                  

B. They were dissatisfied with the curriculum.

C. Other jobs provided higher salaries.          

D. They needed to be retrained

Câu 251 :

The word "inevitably" in paragraph 2 is closest in meaning to____.

A. unavoidably           

B. impartially                

C. irrationally                         

D. unwillingly

Câu 252 :

Which of the following best characterizes the organization of the passage

A. The second paragraph provides a fictional account to illustrate a problem presented in the firstparagraph

B. The second paragraph argues against a point made in the first paragraph.

C. The second paragraph introduces a problem not mentioned in the first paragraph

D. The second paragraph presents the effect of circumstances described in the first paragraph.

Câu 253 :

Read the following passage and mark the letter A, B, C, or D on your answer sheet

to indicate the correct answer to each of the questions.

The cause of tooth decay is acid, which is produced by bacteria in the mouth. The acid removes minerals from tooth enamel, allowing tooth decay to begin; the saliva in your mouth encourages remineralization and neutralizes the acid. The rate at which bacteria in the mouth produce acid depends on the amount of plaque on the teeth, the composition of the microbial flora, and whether the bacteria of the plaque have been "primed" by frequent exposure to sugar. To keep your teeth healthy, a regular dental hygiene program should be followed.

Removing plaque with a toothbrush and dental floss temporarily reduces the numbers of bacteria in the mouth and thus reduces tooth decay. It also makes the surfaces of the teeth more accessible, enabling saliva to neutralize acid and remineralize lesions. If fluoride is present in drinking water when teeth are forming, some fluoride is incorporated into the enamel of the teeth, making them more resistant to attack by acid. Fluoride toothpaste seems to act in another way, by promoting the remineralization of early carious lesions.

In addition to a regular dental hygiene program, a good way to keep your teeth healthy is to reduce your intake of sweet food. The least cavity-causing way to eat sweets is to have them with meals and not between. The number of times you eat sweets rather than the total amount determines how much harmful acid the bacteria in your saliva produce. But the amount of sweets influences the quality of your saliva. Avoid, if you can, sticky sweets that stay in your mouth a long time. Also try to brush and floss your teeth after eating sugary foods. Even rinsing your mouth with water is effective.

Whenever possible, eat foods with fiber, such as raw carrot sticks, apples, celery sticks, etc., that scrape off plaque, acting as a toothbrush. Cavities can be greatly reduced if these rules are followed when eating sweets.

What does this passage mainly discuss?

A. Good nutrition                                            

B. Food with fiber

C. Ways to keep your teeth healthy                

D. Fluoridization and cavities

Câu 254 : The word “it refers to_________.

A. dental floss                        

B. bacteria                     

C. removal of plaque              

D. plaque

Câu 255 :

According to the passage, all of the following statements about plaque are true EXCEPT_________.

A. It consists of acid producing bacteria

B. It is not affected by eating sweets

C. It can be removed from teeth by brushing and flossing

D. It reduces the positive effect of saliva

Câu 256 :
We can infer from the passage that one benefit of fluoride to healthy teeth is_________.

A. It strengthens tooth enamel          

B. It stimulates saliva production

C. It makes teeth whiter                    

D. It is a replacement for brushing and flossing in dental care

Câu 257 :

What can be concluded from the passage about sweets?

A. All sweets should be avoided.

B. Sweets should be eaten with care.

C. It is better to eat sweets a little at a time throughout the day.

D. Sticky sweets are less harmful than other sweets.  

Câu 258 :

The author of the passage states that the amount of acid produced by the bacteria in your saliva increases_________.

A. with the amount of sweets you eat             

B. with the number of times you eat sweets

C. if you eat sweets with your meals               

D. if you eat sticky sweets

Câu 259 : The word "scrape off" is closest in meaning to_________.

A. repel                    

B. rub together with        

C. remove                               

D. dissolve

Câu 260 :

. Read the following passage and mark the letter A, B, C, or D on your answer sheet to choose the best answer for the following questions.

In the past, both men and women were expected to be married at quite young ages. Marriages were generally arranged by parents and family, with their children having little chance to say no in the matter. In the past it was not surprising to find that a bride and groom had only just met on the day of their engagement or marriage.

In modern Vietnam, this has changed completely as people choose their own marriage-partners based on love, and in consideration primarily to their own needs and wants. Moreover early marriage is quite illegal.

The traditional Vietnamese wedding is one of the most important of traditional Vietnamese occasions. Regardless of westernization, many of the age-old customs practiced in a traditional Vietnamese wedding continue to be celebrated by both Vietnamese in Vietnam and overseas, often combining both western and eastern elements. Besides the wedding ceremony, there is also an engagement ceremony which takes place usually half a year or so before the wedding. Due to the spiritual nature of the occasion, the date and time of the marriage ceremony are decided in advance by a fortune teller. The traditional Vietnamese wedding consists of an extensive array of ceremonies: the first is the ceremony to ask permission to receive the bride, the second is the procession to receive the bride (along with the ancestor ceremony at her house), the third is to bring the bride to the groom's house for another ancestor ceremony and to welcome her into the family, then the last is a wedding banquet. The number of guests in attendance at these banquets is huge, usually in the hundreds. Several special dishes are served. Guests are expected to bring gifts, often money, which the groom and bride at one point in the banquet will go from table to table collecting.

 According to the passage, in the past, _____.

A. parents had no right to interfere their children's marriage

B. Vietnamese couples were free to make a decision on the marriage 

C. getting married at an early age was not allowed

D. Vietnamese marriage was decided by parents and family

Câu 261 : In former days, the fact that a bride and groom had only first met just on the day of their engagement or marriage was _________.

A. popular                   

B. uncommon               

C. strange                               

D. surprising

Câu 262 :

Which sentence is referred Vietnamese modern marriage?

A. Couples do not get married at quite young ages.

B. Marriage is quite westernization.

C. Most young people do not have their marriage based on love.

D. All marriages are arranged by parents and family.

Câu 263 :

According to the passage, which of the following is true about the traditional Vietnamese

wedding?

A. Brides’ parents decide the date and time of the marriage ceremony.

B. An engagement ceremony is usually taken place half a year or so before the wedding.

C. Oversea Vietnamese people do not like to organize a traditional wedding.

D. Many of the old-aged customs practiced in a traditional Vietnamese wedding have disappeared.

Câu 264 : The word “array” in paragraph 3 is closest in meaning to _______.

A. line                        

B. process                     

C. group                                 

D. progress

Câu 265 : The word “banquet” in paragraph 3 is closest in meaning to _______.

A. buffet                     

B. big party                   

C. ceremony                           

D. small party

Câu 266 : According to the passge, which does not exist in a Vietnamese wedding party?

A. guests                     

B. dishes                       

C. gifts                                   

D. firecrackers

Câu 267 :

Read the following passage and mark the letter A, B, C or D on your answer sheet to indicate the correct answer to each of the questions.

After two decades of growing student enrollments and economic prosperity, business schools in the United States have started to face harder times. Only Harvard's MBA School has shown a substantial increase in enrollment in recent years. Both Princeton and Stanford have seen decreases in their enrollments. Since 1990, the number of people receiving Masters in Business Administration (MBA) degrees, has dropped about 3 percent to 75,000, and the trend of lower enrollment rates is expected to continue.

There are two factors causing this decrease in students seeking an MBA degree. The first one is that many graduates of four-year colleges are finding that an MBA degree does not guarantee a plush job on Wall Street, or in other financial districts of major American cities. Many of the entry-level management jobs are going to students graduating with Master of Arts degrees in English and the humanities as well as those holding MBA degrees. Students have asked the question, "Is an MBA degree really what I need to be best prepared for getting a good job?" The second major factor has been the cutting of American payrolls and the lower number of entry-level jobs being offered. Business needs are changing, and MBA schools are

struggling to meet the new demands.

 What does the passage mainly discuss?        

A. Types of graduate degrees                          

B. Jobs on Wall Street

C. Changes in enrollment for MBA schools    

D. How schools are changing to reflect the economy

Câu 268 : The underlined word “one” in paragraph 2 most likely refers to

A. MBA degree         

B. decrease                   

C. graduate                            

D. factor

Câu 269 : The underlined word “plush” in paragraph 2 is closest in meaning to which of the following?

A. expensive               

B. affordable                

C. comfortable                       

D. competitive

Câu 270 : Which of the following business schools has NOT shown a decrease in enrollment?

A. Harvard                  

B. Yale                         

C. Stanford                            

D. Princeton

Câu 271 :

The underlined phrase “the trend of” in paragraph 1 could be best replaced by which of the following?

A. the extraction from                                     

B. the advantage of 

C. the movement toward                                 

D. the drawback of

Câu 272 :
hich of the following descriptions most likely applies to Wall Street?

A. a center for international affairs                  

B. a neighborhood in New York 

C. a major financial center                               

D. a shopping district

Câu 273 :

According to the passage, which of the following is a cause of declining business school enrollments?

A. declining population                                   


B. low salary and foreign competition     

C. lack of necessity for an MBA

D. fewer MBA schools

Câu 274 :

Read the following passage and mark the letter A, B, C, or D on your answer sheet to indicate the correct answer to each of the questions.

It is common knowledge that the Internet, often called simply Net, is the largest network of computers in the world. It connects businesses, institutions and individuals. One of its main uses is communication, and electronic mail, or e-mail, is probably the most popular use of the Internet. Although the Net is widely used by people of all ages, many adults are now concerned about is possibly harmful effects on young people.

 example, chatting is a popular activity on the Net. Electronic chat rooms are Internet sites where people can communicate directly with one another. These chat rooms are quite popular with children. However, adults with inappropriate or illegal motives may also join chat rooms. Sometimes these adults try to arrange face-to-face meetings with unsuspecting victims. There have cases where children have been tricked by adults who pass them off as children and who give information that provokes to be entirely false.

Another cause of worry for parents is the huge amount of information accessible to users of the Internet. Not all information on the Net is accurate, and some deliberately misleading. In addition, parents worry that children may be exposed to harmful material. For instance, there are innumerable sites where explicitly violent or pornographic scenes are shown. Moreover, many organizations maintain hate sites that present racist or politically extreme views.

The Internet has tremendous potential for good, but it has also led to some serious questions about appropriate use. Where protection of their children is needed, parents should not hesitate to intervene. Unlimited freedom on the Net can lead to serious, sometimes dangerous, consequences.

 The author generally believes that children _____.

A. can learn a great deal from the Net                        

B. should never be allowed to use the Net

C. should not have unsupervised access to the Net    

D. should be allowed to use the Net today

Câu 275 :

According to the first paragraph, which group is most likely to be worried about children’s use of the Internet?

A. parents        

B. teenagers    

C. business people      

D. People who work in institutions

Câu 276 :

In chat rooms, _____.

A. people communicate face-to-face             

B. people may give a false impression

C. you know who you are talking to             

D. parents and children communicate with each other

Câu 277 : The expression pass themselves off as in paragraph 2 means ____.

A. look like                 

B. participate                

C. pretend to be                     

D. contribute

Câu 278 :

What can we understand about the face-to-face meetings mentioned in paragraph 2?

A. All people in chat groups have them.

B. They may turn out to be harmful to children.

C. They help to communicate better.

D. They give children false information.

Câu 279 :

The racist and political extreme views mentioned in paragraph 3 are example of _____.

A. what you see on some Internet sites 

B. what children think they use the Internet 

C. parental intervention 

D. the huge amount of information available.

Câu 280 :

We can understand from the passage that _____.

A. there is not enough supervision on the Internet   

B. politicians control the Internet

C. parents should not intervene                                

D. the Internet leads to criminal activity

Câu 281 :

It seems that the main point the author is trying to make about the Internet is that it _____.

A. can be dangerous                                                     

B. provides unlimited access to information 

C. is beneficial                                                          

D. should only be used by adults

Câu 282 :

Read the following passage and mark the letter A, B, C or D on your answer sheet to indicate the correct answer to each of the following questions.

Plants are subject to attack and infection by a remarkable variety of symbiotic species and have evolved a diverse array of mechanisms designed to frustrate the potential colonists. These can be divided into preformed or passive defense mechanisms and inducible or active systems. Passive plant defense comprises physical and chemical barriers that prevent entry of pathogens, such as bacteria, or render tissues unpalatable or toxic to the invader. The external surfaces of plants, in addition to being covered by an epidermis and a waxy cuticle, often carry spiky hairs known as trichomes, which either prevent feeding by insects or may even puncture and kill insect larvae. Other trichomes are sticky and glandular and effectively trap and immobilize insects. If the physical barriers of the plant are breached, then preformed chemicals may inhibit or kill the intruder, and plant tissues contain a diverse array of toxic or potentially toxic substances, such as resins, tannins, glycosides, and alkaloids, many of which are highly effective deterrents to insects that feed on plants. The success of the Colorado beetle in infesting potatoes, for example, seems to be correlated with its high tolerance to alkaloids that normally repel potential pests. Other possible chemical defenses, while not directly toxic to the parasite, may inhibit some essential step in the establishment of a parasitic relationship. For example, glycoproteins in plant cell walls may inactivate enzymes that degrade cell walls. These enzymes are often produced by bacteria and fungi.

Active plant defense mechanisms are comparable to the immune system of vertebrate animals, although the cellular and molecular bases are fundamentally different. Both, however, are triggered in reaction to intrusion, implying that the host has some means of recognizing the presence of a foreign organism. The most dramatic example of an inducible plant defense reaction is the hypersensitive response. In the hypersensitive response, cells undergo rapid necrosis ― that is, they become diseased and die ― after being penetrated by a parasite ; the parasite itself subsequently ceases to grow and is therefore restricted to one or a few cells around the entry site. Several theories have been put forward to explain the bases of hypersensitive resistance.

What does the passage mainly discuss?

A. How the immune system of animals and the defense mechanisms of plants differ 

B. How plant defense mechanisms function

C. Theories on active plant defense mechanisms

D. The success of parasites in resisting plant defense mechanisms

Câu 283 : The phrase "subject to" in line 1 is closest in meaning to

A. attractive to           

B. strengthened by       

C. classified by                      

D. susceptible to

Câu 284 : The word "puncture" in line 8 is closest in meaning to

A. surround                

B. pinch                        

C. pierce                                 

D. cover

Câu 285 : The word "which" in line 13 refers to

A. barriers                   

B. tissues                       

C. substances                         

D. insects

Câu 286 : Which of the following substances does the author mention as NOT necessarily being toxic to the Colorado beetle?

A. Alkaloids                

B. Glycosides               

C. Resins                                

D. Tannins

Câu 287 :

Why does the author mention "glycoproteins" in line 17?

A. To illustrate how chemicals function in plant defense

B. To emphasize the importance of physical barriers in plant defense

C. To compare plant defense mechanisms to the immune system of animals 

D. To introduce the discussion of active defense mechanisms in plants

Câu 288 : Where in the passage dose the author describe an active plant-defense reaction?

A. Lines 1-3                

B. Lines 13-15              

C. Lines 19-23                       

D. Lines 4-6

Câu 289 :

The passage most probable continues with a discussion of theories on

A. the basis of passage plant defense.             

B. the principles of the hypersensitive response.

C. how chemicals inhibit a parasitic relationship.       

D. how plants produce toxic chemical.

Câu 290 :

Read the following text and mark the letter A, B, C or D on your answer sheet to indicate the correct answer to each of the questions.

Many ants forage across the countryside in large numbers and undertake mass migrations; these activities proceed because one ant lays a trail on the ground for the others to follow. As a worker ant returns home after finding a source of food, it marks the route by intermittently touching its stinger to the ground and depositing a tiny amount of trail pheromone – a mixture of chemicals that delivers diverse messages as the context changes. These trails incorporate no directional information and may be followed by other ants in either direction.

Unlike some other messages, such as the one arising from a dead ant, a food trail has to be kept secret from members of other species. It is not surprising then that ant species use a wide variety of compounds as trail pheromones. Ants can be extremely sensitive to these signals. Investigators working with the trail pheromone of the leafcutter ant Atta texana calculated that one milligram of this substance would suffice to lead a column of ants three times around Earth.

The vapor of the evaporating pheromone over the trail guides an ant along the way, and the ant detects this signal with receptors in its antennae. A trail pheromone will evaporate to furnish the highest concentration of vapor right over the trail, in what is called a vapor space. In following the trail, the ant moves to the right and left, oscillating from side to side across the line of the trail itself, bringing first one

and then the other antenna into the vapor space. As the ant moves to the right, its left antenna arrives in the vapor space.

The signal it receives causes it to swing to the left, and the ant then pursues this new course until its right antenna reaches the vapor space. It then swings back to the right, and so weaves back and forth down the trail.

What does the passage mainly discuss?

A. The information contained in pheromones           

B. The mass migration of ants

C. How ants mark and follow a chemical trail          

D. Different species of ants around the world

Câu 291 : The word “intermittently” in paragraph 1 is closest in meaning to

A. roughly                  

B. incorrectly                

C. rapidly                               

D. periodically

Câu 292 : The phrase “the one” in paragraph 2 refers to a single

A. message                 

B. food trail                  

C. dead ant                            

D. species

Câu 293 :

According to the passage, why do ants use different compounds a trail pheromones?

A. To reduce their sensitivity to some chemicals 

B.To indicate how far away the food is 

C. To attract different types of ants 

D. To protect their trail from other species

Câu 294 :
The author mentions the trail pheromone of the leafcutter ant in paragraph 2 to point out

A. a type of ant that is common in many parts of the world

B. how little pheromone is needed to mark a trail

C. the different types of pheromones ants can produce

D. that certain ants can produce up to one milligram of pheromone

Câu 295 : According to the passage, how are ants guided by trail pheromones?

A. They sense the vapor through their antennae.

B. They avoid the vapor spaces by moving in a straight line.

C. They concentrate on the smell of food.

D. They follow an ant who is familiar with the trail

Câu 296 : According to the passage, the highest amount of pheromone vapor is found

A. in the source of food                                   

B. in the receptors of the ants

C. under the soil along the trail                       

D. just above the trail

Câu 297 :

Read the following passage and mark the letter A, B, C, or D on your answer sheet to indicate the correct answer to each of the questions.

On the fourth Thursday in November, in houses around the United States, families get together for a feast, or a large meal. Almost all families eat turkey and cranberry sauce for this meal, and have pumpkin pie for dessert. This feast is part of a very special day, the holiday of Thanksgiving.

In 1620 the Pilgrims made a difficult trip across the ocean from England. They landed in what is now Massachusetts. In England the Pilgrims had not been allowed to freely practice their religion. So they went to the New World in search of religious freedom.

The Pilgrims' first winter was very hard. Almost half the group died of cold, hunger and disease. But the Indians of Massachusetts taught the Pilgrims to plant corn, to hunt and to fish. When the next fall came, the Pilgrims had plenty of food. They were thankful to God and the Indians and had a feast to give thanks. They invited the Indians to join them. This was the first Thanksgiving.

Thanksgiving became a national holiday many years later because of the effort of a woman named Sarah Hale. For forty years Sarah Hale wrote to each president and asked for a holiday of Thanksgiving. At last she was successful. In 1863 President Lincoln declared Thanksgiving a holiday.

How much is Thanksgiving today like the Pilgrims’ Thanksgiving? In many ways they are different. For example, historians think that the Pilgrims ate deer, not turkey. The idea of Thanksgiving, though, is very much the same: Thanksgiving is a day on which we celebrate and give thanks.

When did the the Pilgrims make a difficult trip to across the ocean from England?

A. in 1863                  

B. in 1621       

C. in 1830                              

D. in 1620

Câu 298 :
The Pilgrims immigrated to the New World because _____.

A. They wanted to search for religious freedom.

B. They wanted to be taught how to plant corn.

C. They wanted to have more land to cultivate.

D. They wanted to make a difficult trip.

Câu 299 : According to the passage, today’s Thanksgiving _____.

A. is only celebrated in Massachustets

B. is a day on which the Pilgrims eat deer

C. is different from the Pilgrims’s Thanksgiving in many ways

D. is just like the Pilgrims’s Thanksgiving

Câu 300 : Which of the following is NOT true about Thanksgiving?

A. It is celebrated on the fourth Thursday on November.

B. It is a day on which Americans celebrate and give thanks.

C. Americans usually have turkey, cranberry sauce and pumpkin pie for this occasion.

D. It became a national holiday thanks to President Lincoln’s 40-year efforts.

Câu 301 : The word “they” in paragraph 3 refer to _____.

A. families                  

B. the Pilgrims              

C. thanks                      

D. the Native Americans

Lời giải có ở chi tiết câu hỏi nhé! (click chuột vào câu hỏi).

Copyright © 2021 HOCTAP247